Neuro CNS m. billeder Flashcards

1
Q

Du sidder i øjenambulatoriet og skal se en tidligere øjenrask 47-årig mand, som er henvist fra endokrinologisk afdeling grundet problemer med synsfeltet. Patienten har fået lavet en perimetriundersøgelse.

Sæt kryds ved det rigtige udsagn:
a. Patienten har en bitemporal hemianopsi, og læsionen sidder ved
chiasma opticum
b. Patienten har en binasal hemianopsi, og læsionen sidder i
occipitallapperne
c. Patienten har en homonym hemianopsi, og læsionen sidder i venstre n.
opticus
d. Patienten har en homonym hemianopsi og læsionen sidder ved chiasma
opticum
e. Patienten har en homonym hemianopsi og læsionen sidder i højre n.
opticus

A

a. Patienten har en bitemporal hemianopsi, og læsionen sidder ved
chiasma opticum

How well did you know this?
1
Not at all
2
3
4
5
Perfectly
2
Q

Hvilket af nedenstående symptomer er mest sandsynligt, hvis et vestibularis
schwannom vokser sig stort?
a. Dobbeltsyn
b. Udtalebesvær
c. Hæshed
d. Ændret ansigtssensibilitet
e. Blindhed

A

d. Ændret ansigtssensibilitet

How well did you know this?
1
Not at all
2
3
4
5
Perfectly
3
Q

Hvad forstår man ved vestibularis- schwannom?

a. En benign tumor udgående fra støttecellerne omkring 7. hjernenerve
b. En benign tumor i porus acusticus externus
c. En malign tumor i den cerebellopontine vinkel
d. En benign tumor udgående fra schwannske støtteceller
e. En malign tumor udgående fra dura

A

d. En benign tumor udgående fra schwannske støtteceller

How well did you know this?
1
Not at all
2
3
4
5
Perfectly
4
Q

En 65-årig kvinde indlægges efter flere ugers tiltagende problemer med at tale,
tygge og synke. På det seneste har man bemærket at hun ikke kan holde hovedet
oppe og at hendes øjenlåg hænger. Generne udviser en vis variation i intensitet og
da det hele startede var der et tydeligt døgnmønster, idet generne klart forværredes
opad dagen. Da du modtager patienten er hun så snøvlende, at du faktisk ikke kan
forstå hvad hun siger. Hun savler ud af munden og hendes ene øjenlåg er helt
tillukket, mens det andet dækker halvdelen af pupillen. Hun har divergerende
øjenakser og ved den objektive undersøgelse finder du utvetydig udtrætning af
musklerne i øjne og arme. Hendes arteriegasser er normale. Hendes
respirationsfrekvens er 34 per minut. Hvilke af nedenstående udsagn er korrekt?
a. Sålænge arteriegasserne er i orden er patienten ikke respiratorisk truet
b. Uanset hvad det drejer sig om, er der ingen grund til at indlede aktiv
behandling, da patientens langstidsprognose tydeligvis er yderst dyster
c. Der findes ikke nogen enkelblodprøve, som kan være med til at stille
diagnosen
d. Patienten er respiratorisk truet trods normale arteriegasser, og der er
også i risiko for, at hun aspirerer

A

d. Patienten er respiratorisk truet trods normale arteriegasser, og der er
også i risiko for, at hun aspirerer

How well did you know this?
1
Not at all
2
3
4
5
Perfectly
5
Q

Du er læge på neurologisk afdeling. På afdelingen ligger en 52 årig tidligere
rask mand, som blev indlagt for 1 døgn siden med venstresidig homonym
hemianopsi, venstresidig central facialisparese, venstresidig hemiparalyse og
neglect. CTC viste stort højresidigt mediainfarkt. Her til morgen er patienten blevet
dårligere. Klokken er nu 8:25 og du finder patienten bevidsthedspåvirket, han åbner
lige akkurat øjne på smertestimuli, svarer ikke på tiltale og den højre pupil er større
end venstre.
Hvad er din tentative diagnose?
a. Malignt mediainfarkt
b. Træthed efter at være blevet vækket til blodtrykskontrol hver 1/2 time
natten igennem
c. Symptomatisk epilepsi
d. Nyt infarkt, nu i venstre hemisfære

A

a. Malignt mediainfarkt

How well did you know this?
1
Not at all
2
3
4
5
Perfectly
6
Q

En 89-årig kvinde falder om på gaden med hjertestop. Der er folk på stedet, der
opstarter hjerte-lungeredning med det samme. Efter nogle minutters hjertemassage
og stød fra en hjertestarter genoplives hun.
Hvilken af de nedenstående typer af nerveceller har med størst sandsynlighed taget
skade af den kortvarige globale iskæmiske påvirkning?
a. Alfa-motoneuroner i rygmarvens forhorn
b. Neuroendokrine nerveceller i nucleus paraventricularis i hypothalamus
c. Pyramideceller i CA1 regionen af hippocampus og Purkinjeceller i
cerebellum

A

c. Pyramideceller i CA1 regionen af hippocampus og Purkinjeceller i
cerebellum

How well did you know this?
1
Not at all
2
3
4
5
Perfectly
7
Q

69 årig tidligere rask mand, der aldrig har været behandlet med fast medicin,
indlægges akut som trombolyse kandidat i Fælles Akutmodtagelsen Odense
Universitetshospital Kl 12 har han talt med pårørende og var uden symptomer. Kl 14
da pårørende kommer på besøg har patienten nedsat kraft over højre arm og ben og
problemer med at tale tydeligt.
Ved ankomsten til Fælles Akutmodtagelsen noteres i journalen højresidig
hemiparalyse og let ekspressiv afasi. BT 140/85. Puls 134 uregelmæssig. Pt vågen
og klar.
På mistanke om cerebral apopleksi foretages akut CT-cerebrum som viser:
Stor blødning i venstre hemisfære med gennembrud til ventrikelsystemet. Der er ikke
blod i fissura sylvii.
Hyppigste årsag til intracerebral blødning er?
a. Bristet aneurysme
b. Forhøjet blodtryk
c. Atrieflimren
d. Koagulopati
e. Stofmisbrug

A

b. Forhøjet blodtryk

How well did you know this?
1
Not at all
2
3
4
5
Perfectly
8
Q

En 70-årig kvinde indlægges på neurologisk afdeling pga akut opståede
synsproblemer. Ved neurologisk undersøgelse findes som det eneste neurologiske
udfald nedenfor viste synsfeltdefekt, der er fundet ved almindelig klinisk
synsfeltundersøgelse. Defekten er angivet som det sorte område.
Hvilken struktur i nervesystemet er læderet?
a. Chiasma opticum
b. Højre occipitallap
c. Højre temporallap
d. Venstre occipitallap
e. Venstre temporallap

A

b. Højre occipitallap

How well did you know this?
1
Not at all
2
3
4
5
Perfectly
9
Q

En 35-årig mand henvises med en venstresidig ansigtslammelse. Lammelsen
udviklede sig over et par dage for ca. en uge siden. Ved undersøgelse finder du, at
der er påvirkning af panderynkning, øjenlukning og mundens bevægelser på venstre
side. På forespørgsel oplyser han, at han havde et rødt udslæt på venstre underben
for 2 måneder siden. Det så mærkeligt ud og han tog et billede med sin iPhone.
Udslættet forsvandt af sig selv og han var ikke til lægen i den anledning.
Hvad vil du foretage dig?
a. Bestille en akut CT-scanning af hjernen, da der formentlig er tale om en
blodprop eller en blødning i højre hemisfære
b. Hurtigst muligt få udført en lumbalpunktur med henblik på undersøgelse
af spinalvæsken
c. Bestille en MR-scanning af hjernen, da der må være tale om en læsion i
hjernestammen
d. Give kombination af binyrebarkhormon og acyclovir, da det med stor
sandsynlighed er en herpesinfektion

A

b. Hurtigst muligt få udført en lumbalpunktur med henblik på undersøgelse

How well did you know this?
1
Not at all
2
3
4
5
Perfectly
10
Q

En 54 årig tidligere rask kvinde henvender sig til egen læge. Hun har nu
gennem 3 uger haft næsten daglig optrædende højresidig dunkende hovedpine
ledsaget af kvalme og opkastninger. Hovedpinen forværres ved fysisk aktivitet. Hun
fortæller, at hendes mor, mormor og søster lider af svær migræne. Objektivt findes
venstresidig hyperrefleksi og venstresidig Babinskis tåfænomen, men herudover er
der normale neurologiske forhold.
Hvilken af følgende diagnoser vil du mistænke?
a. Hortons hovedpine
b. Migræne med aura
c. Spændingshovedpine
d. Migræne uden aura
e. Tumor cerebri

A

e. Tumor cerebri

How well did you know this?
1
Not at all
2
3
4
5
Perfectly
11
Q

En artikel i JAMA Pediatrics sidste år (2015;169:822-829) anvendte MR
scanning af hjernen fra 389 amerikanske børn i en prospektiv undersøgelse. Børn
fra familier med lav indkomst (under den officielle fattigdomsgrænse) havde 8-10%
lavere volumen af grå substans, og de scorede 4-7 IQ point lavere i standardiserede
test, begge dele statistisk signifikant (p ‹ 0,05).
Kan man konkludere, at lav indkomst er en risikofaktor for nedsat hjerneudvikling? Et
af følgende svar er forkert – hvilket?
a. Nej, hvis fattigdom virkelig spillede så stor en rolle, ville indkomst have
langt større virkning på fordelingen af intelligens i samfundet
b. Ja, i den forstand at lavere volumen af grå substans er korreleret med IQ,
og begge sammenhænge peger i samme retning
c. Nej, indkomst som sådan er ikke relevant, det er derimod de forskellige
risici, som følger med fattigdom, herunder dårlig ernæring og manglende
stimulation
d. Ja, for lav indkomst er også forbundet med andre risici, så som
alkoholisme hos moderen, stofmisbrug og andre eksponeringer for
neurotoksiske stoffer
e. Nej, for det fremgår ikke, om der er korrigeret for andre faktorer, herunder arvelige, der kan påvirke hjernens udvikling

A

a. Nej, hvis fattigdom virkelig spillede så stor en rolle, ville indkomst have
langt større virkning på fordelingen af intelligens i samfundet

How well did you know this?
1
Not at all
2
3
4
5
Perfectly
12
Q

En 73-årig mand er kendt med atrieflimren. Han er i digoxinbehandling og i
antikoagulationsbehandling med Marevan (K-vitaminantagonist). Han indlægges nu
akut i neurologisk afdeling på grund af pludselig opstået afasi og højresidig
hemiparese. CT-skanning af cerebrum ses ovenfor og viser:
a. Kronisk subduralt hæmatom
b. Tumor cerebri
c. Cerebralt infarkt
d. Haemorragia cerebri
e. Epiduralt hæmatom

A

c. Cerebralt infarkt

How well did you know this?
1
Not at all
2
3
4
5
Perfectly
13
Q

På neurokirurgisk afdeling indlægges en 68-årig mand med akut intracerebral
blødning i højre cerebrale hemisfære.
Hvilken del af hjernen er ikke i risiko for at herniere pga. det øgede intrakranielle
tryk?
a. Tonsilla cerebelli
b. Thalamus
c. Gyrus cinguli
d. Uncus i den mediale del af temporallappen

A

b. Thalamus

How well did you know this?
1
Not at all
2
3
4
5
Perfectly
14
Q

72-årig mand med 2 tilfælde af 15 minutters varende let afasi og højresidig
armparese. Han tager ikke noget medicin og har ikke noget alkoholoverforbrug.
Udsnit af hans EKG ser således ud:
Hvilken sekundær profylaktisk behandling bør vælges til denne patient?
a. Magnyl og dipyridamol (persantin)
b. Vitamin K-antagonist (Marevan)
c. Magnyl
d. Digoxin og magnyl
e. Magnyl, dipyridamol (persantin) og vitamin K-antagonist (Marevan)

A

b. Vitamin K-antagonist (Marevan)

How well did you know this?
1
Not at all
2
3
4
5
Perfectly
15
Q

I neurologisk ambulatorium ser du en 24-årig tidligere rask kvinde uden kendte arvelige sygdomme i familien. Hun opsøgte første gang egen læge for 12 måneder siden pga. ændring af farvesansen (blå /grøn) og samtidig udvikling af smerter bag højre øje og nedsat syn på højre øje. Symptomerne forsvandt uden yderligere udredning eller behandling i løbet af 2 uger. For 6 måneder siden havde kvinden 1 uge varende dobbeltsyn ved blik mod højre, men da hun var på Tasmanien på ferie og dobbelt synet forsvandt spontant opsøgte hun ikke læge. Grunden til den aktuelle
konsultation er 1 uge varende snurrende fornemmelse i hænderne.
Objektivt finder du følgende: Ved ofthalmoskopi let afbleget papil på højre øje og diskret insufficiens af m. abducens ved sideblik mod højre. Der er desuden hyperrefleksi svarende til biceps og patellar reflekser, samt lette sensoriske abnormiteter i begge håndflader (dynamisk mekanisk allodyni).
Hvilket udsagn er korrekt?
a. Sygehistorien samt de objektive fund tyder på ALS og hun bør udredes
med MR scanning af collumna cervicalis og ENG undersøgelse
b. Sygehistorien og de objektive fund tyder på multipel sklerose. Undertypen
er primært progressiv (PPMS). Hun skal udredes med MR scanning af
hjernen samt undersøgelse af rygmarvsvæsken
c. Sygehistorien og de objektive fund tyder på multipel sklerose.
Undertypen er attakvis (RRMS). Hun udredes med MR scanning af hjernen
og medulla samt undersøgelse af rygmarvsvæsken
d. Sygehistorien og de objektive fund tyder på multipel sklerose. Undertypen
er sekundær progressiv (SPMS). Hun udredes med MR scanning af hjernen
samt undersøgelse af rygmarvsvæsken

A

c. Sygehistorien og de objektive fund tyder på multipel sklerose.
Undertypen er attakvis (RRMS). Hun udredes med MR scanning af hjernen

How well did you know this?
1
Not at all
2
3
4
5
Perfectly
16
Q

En 49- årig mand arbejder som tømrer og falder i den forbindelse ned fra et stillads
og slår hovedet. Han er kortvarig bevidstløs, men vækkes hurtigt og svarer relevant
på spørgsmål. Han er orienteret i tid, sted og egne data.
Ved den objektive undersøgelse konstateres det, at hans højre mundvig hænger,
når han forsøger at smile.
Mistanken går på, at han ved faldet har pådraget sig en højresidig perifer
facialisparese, som medfører, at højre mundvig ikke kan bevæges.
Hvilket af nedenstående fund vil bekræfte mistanken om perifer højresidig
facialisparese på denne patient?
a. Patienten kan ikke rynke panden på nogen af siderne
b. Patienten kan ikke rynke panden i venstre side
c. Patienten kan ikke rynke panden i højre side
d. Patienten kan rynke panden i begge sider
e. Venstresidig hemiplegi

A

c. Patienten kan ikke rynke panden i højre side

How well did you know this?
1
Not at all
2
3
4
5
Perfectly
17
Q

En 49-årig mand arbejder som tømrer og falder i den forbindelse ned fra et stillads og slår hovedet. Han er kortvarig bevidstløs, men vækkes hurtigt og svarer relevant på spørgsmål. Han er orienteret i tid, sted og egne data.
Ved den objektive undersøgelse konstateres det, at hans højre mundvig hænger, når han forsøger at smile.
I forbindelse med den objektive undersøgelse konstateres det, at udover traumatisk betinget højresidig perifer facialisparese er der blod i højre øregang. Klar væske siver desuden fra det højre øre.
Hvilket af nedenstående tyder fundene på?
a. Patienten har pådraget sig en traumatisk hjerneblødning
b. Patienten har pådraget sig en fraktur involverende os temporale
c. Patienten har pådraget sig en perilymfatisk fistel
d. Patienten har pådraget sig et barotraume
e. Patienten har pådraget sig et othæmatom

A

b. Patienten har pådraget sig en fraktur involverende os temporale

How well did you know this?
1
Not at all
2
3
4
5
Perfectly
18
Q

Du vurderer at han med overvejende sandsynlighed har pådraget sig en fraktur involverende os temporale.
Der var glucose i den klare væske fra øret og du vurderer at patienten har
otoliquorrhoea. Hvilken behandling heraf er mest relevant indenfor de første 48 timer?
a. Lukning af lækagen
b. Anlæggelse af spinaldræn
c. Dekompression af lobus frontalis
d. Obliteration af sinus frontalis
e. Evakuering af patientens intrakranielle hæmatom

A

b. Anlæggelse af spinaldræn

How well did you know this?
1
Not at all
2
3
4
5
Perfectly
19
Q

Du har stor mistanke om, at patienten har pådraget sig en fraktur involverende os temporale.
CT scanning af kraniet bekræfter at der foreligger fraktur i os temporale dexter. Audiometri viser både konduktiv og sensorineural hørenedsættelse på højre side ifølge audiologiassistenten. Hvilket af følgende alternativer forventer du derfor at finde i audiogrammet?
a. Højre øres lydledningstærskler og knogleledningstærskler er ens
b. Højre øres lydledningstærskler er bedre end højre øres
knogleledningstærskler
c. Højre øres knogleledningstærskler er bedre end højre øres
lydledningstærskler
d. Højre øres knogleledningstærskler er bedre end venstre øres
knogleledningstærskler
e. Højre øres lydledningstærskler er bedre end ventre øres
lydledningstærskler

A

c. Højre øres knogleledningstærskler er bedre end højre øres
lydledningstærskler

How well did you know this?
1
Not at all
2
3
4
5
Perfectly
20
Q

En 49- årig mand arbejder som tømrer og falder i den forbindelse ned fra et stillads og slår hovedet. CT-scanning af kraniet viser fraktur i os temporale dexter. Patienten klager efter nogle dage over, at han synes at maden smager af mindre på højre side af tungen end den plejer. Hvad er den mest sandsynlige forklaring på dette?

a. Læsion af n.facialis
b. Læsion af n.glossopharyngeus
c. Læsion af n.vagus
d. Læsion af n.trigeminus
e. Læsion af nervus hypoglossus

A

a. Læsion af n.facialis

How well did you know this?
1
Not at all
2
3
4
5
Perfectly
21
Q

En 31-årig mandlig bilist indbringes til akutmodtagelsen efter natligt ulykkestilfælde, hvor han som bilist med høj hastighed tørner ind i et vejtræ. Lægeambulancen er hurtigt på stedet, og finder manden bevidstløs bag rattet. Man ser straks et stort subkutant hæmatom i højre side af panden og ned over højre øje. Da du kommer til skadestuen, er patienten allerede gennemgået og undersøgt af de øvrige relevante specialer. Patienten responderer ikke på tiltale eller stimulation.
Som vagthavende neurokirurg gør du et af følgende:
a. Foretager hurtig klinisk vurdering med ophtalmoskopi (papilødem?),
motorisk undersøgelse og samt refleksundersøgelse.
Der foretages derefter akut MR-skanning af hjernen.
b. Du foretager objektiv klinisk vurdering mhp på kranie-og spinal traume,
scoring i henhold til Glasgow Coma Scale Score, og finder at patienten scorer 0. Han intuberes og køres til akut CT-skanning af hjernen
c. Du foretager objektiv klinisk vurdering mhp på kranie-og spinal traume, scoring i henhold til Glasgow Coma Scale Score, og finder at patienten scorer 3. Han intuberes og køres til akut CT-skanning af hjernen.
d. Pt. indlægges, og der bestilles CT-skanning med kontrastindgift.
e. Pt. Intuberes og køres straks til CT-skanning af hjernen med
kontrastindgift.

A

c. Du foretager objektiv klinisk vurdering mhp på kranie-og spinal traume, scoring i henhold til Glasgow Coma Scale Score, og finder at patienten scorer 3. Han intuberes og køres til akut CT-skanning af hjernen.

How well did you know this?
1
Not at all
2
3
4
5
Perfectly
22
Q

En 31-årig mandlig bilist indbringes til akutmodtagelsen efter natligt ulykkestilfælde, hvor han som bilist med høj hastighed tørner ind i et vejtræ. Lægeambulancen er hurtigt på stedet, og finder manden bevidstløs bag rattet. Man ser straks et stort subkutant hæmatom i højre side af panden og ned over højre øje. Da du kommer til skadestuen, er patienten allerede gennemgået og undersøgt af de øvrige relevante specialer. Patienten responderer ikke på tiltale eller stimulation.

Patienten har nu fået udført CT-scanning af hjernen.
CT-skanningen viser et større akut subduralt hæmatom (ASH) (fig. 1)
a. ASH er mindre hyppige og opstår ved alt fra lette til svære hovedtraumer.
De har generelt en god prognose og kræver sjældent behandling
b. ASH opstår typisk ved lette til moderate hovedtraumer og har hyppigt en
klinisk latensperiode før symptomerne sætter ind (“talk and die”)
c. ASH opstår typisk ved lette til moderate hovedtraumer og har hyppigt en
klinisk latensperiode før symptomerne sætter ind (“talk and die”)
d. ASH opstår ved moderate til svære hovedtraumer og medfører ofte akut
bevidsthedspåvirkning og forhøjet intrakranielt tryk. Der kan foretages
lumbalpunktur, som ofte viser det forhøjede tryk og i sådanne tilfælde
overflyttes patienten altid til neurokirurgisk afdeling
e. ASH opstår ved moderate men oftest svære hovedtraumer og kan
medføre akut indsættende bevidstløshed. Patienterne udvikler som regel
forhøjet intrakranielt tryk, som kan kræve operativ aflastning

A

e. ASH opstår ved moderate men oftest svære hovedtraumer og kan
medføre akut indsættende bevidstløshed. Patienterne udvikler som regel
forhøjet intrakranielt tryk, som kan kræve operativ aflastning

How well did you know this?
1
Not at all
2
3
4
5
Perfectly
23
Q

En 31-årig mandlig bilist indbringes til akutmodtagelsen efter natligt ulykkestilfælde, hvor han som bilist med høj hastighed tørner ind i et vejtræ. Lægeambulancen er hurtigt på stedet, og finder manden bevidstløs bag rattet. Man ser straks et stort subkutant hæmatom i højre side af panden og ned over højre øje. Da du kommer til skadestuen, er patienten allerede gennemgået og undersøgt af de øvrige relevante specialer. Patienten responderer ikke på tiltale eller stimulation.

Efter indlæggelse implanteres en intrakraniel trykmåler ved at skue en lille bolt ind i
kraniet og føre et målekateter lidt ned i hjerneparenkymet.
a. Den intrakranielle tryk/volumen-kurve, dvs. det intrakranielle tryk (ICP)
som funktion af intrakranielt ekstra-volumen (som fx hæmatom og/eller ødem)
er en næsten lineær funktion som fluktuerer med puls og respiration
b. Tryk/volumen-kurven er bemærkelsesværdigt stabil og varierer kun
beskedent efter volumenøgninger, da det intrakranielle kompartment har en
betydelig bufferkapacitet (“compliance”)
c. ICP ændres kun beskedent ved mindre volumenøgninger, da man har
bufferkapacitet ved at foretage hyperventilation og drænere
cerebrospinalvæske (CSF), fx ved lumbalpunktur
d. ICP ændres kun beskedent ved mindre volumenøgninger, da man har
bufferkapacitet ved mindskning af CSF og blodvolumen. Ved
volumenøgninger over 50-80 ml stiger ICP markant
e. Normalt ICP er 20-25 mmHg, men ved hurtige intrakranielle
volumenøgninger, som fx ved hæmatomdannelse, kan ICP stige til op mod
80 mmHg inden symptomer på forhøjet ICP kan registreres

A

d. ICP ændres kun beskedent ved mindre volumenøgninger, da man har
bufferkapacitet ved mindskning af CSF og blodvolumen. Ved
volumenøgninger over 50-80 ml stiger ICP markant

How well did you know this?
1
Not at all
2
3
4
5
Perfectly
24
Q

En 31-årig mandlig bilist indbringes til akutmodtagelsen efter natligt ulykkestilfælde, hvor han som bilist med høj hastighed tørner ind i et vejtræ. Lægeambulancen er hurtigt på stedet, og finder manden bevidstløs bag rattet. Man ser straks et stort subkutant hæmatom i højre side af panden og ned over højre øje. Da du kommer til skadestuen, er patienten allerede gennemgået og undersøgt af de øvrige relevante specialer. Patienten responderer ikke på tiltale eller stimulation.

Det intrakranielle tryk kan mindskes ved:

a. Hypoventilation pga CO2-ophobning
b. Hypoventilation pga CO2-mindskning
c. Hyperventilation pga CO2-mindskning
d. Hyperventilation pga CO2-ophobning

A

c. Hyperventilation pga CO2-mindskning

How well did you know this?
1
Not at all
2
3
4
5
Perfectly
25
Q

Forhøjet ICP kan også opstå på ikke-traumatisk basis ved fx. hydrocefalus.
a. Hydrocefalus skyldes i reglen CSF-ophobning pga defekt resorption i 4.
ventrikel
b. Hydrocefalus kan inddeles i 2 hovedgrupper: obstruktiv eller
kommunicerende. CSF-resorptionen foregår i 4. ventrikel
c. Medfødt hydrocefalus er i reglen kommunicerende og kan ses uden og i
kombination med myelomeningocele
d. Medfødt hydrocefalus er i reglen obstruktiv og kan ses uden og i
kombination med myelomeningocele. Resorptionsdefekten sidder i sinus
sagittalis superior
e. Hydrocefalus kan inddeles i 2 hovedgrupper: Obstruktiv eller
kommunicerende. CSF-resorptionen foregår primært i sinus sagittalis
superior

A

e. Hydrocefalus kan inddeles i 2 hovedgrupper: Obstruktiv eller
kommunicerende. CSF-resorptionen foregår primært i sinus sagittalis
superior

How well did you know this?
1
Not at all
2
3
4
5
Perfectly
26
Q

Du arbejder i skadestuen og bliver bedt om at tilse en 29 årig kvinde. Hun kommer da hun igennem de seneste dage har fået fornemmelse af tiltagende svigt af venstre ben. Hun er tidligere rask og får ingen medicin fraset p-piller. Hun har dog for ca. 2 år siden været set af en øjenlæge grundet sløret syn på højre øje. Disse symptomer kom også gradvist men fortog sig indenfor 2 uger.
Du finder følgende symptomer:
 Går med let efterslæb af venstre ben
 Kraft ved hoftefleksion på venstre side MRC score 4, kraft ved knæfleksion på
venstre side MRC score 4 og kraft ved dorsalfleksion af venstre fodled MRC
score 3
 Hyperrefleksi i begge ben med bilateral Babinskis tåfænomen og fodklonus på
venstre side
 Nedsat vibrationssans på venstre ben til knæniveau og på højre ben til
ankelniveau
Hvilken sygdom vil være den mest sandsynlige baseret på anamnesen og de objektive fund?
a. Guillain Barre syndrom
b. Apopleksia cerebri
c. Lumbal discusprolaps
d. Multipel Sklerose
e. Medullær tværsnitssyndrom

A

d. Multipel Sklerose

How well did you know this?
1
Not at all
2
3
4
5
Perfectly
27
Q

Du arbejder i skadestuen og bliver bedt om at tilse en 29 årig kvinde. Hun kommer da hun igennem de seneste dage har fået fornemmelse af tiltagende svigt af venstre ben. Hun er tidligere rask og får ingen medicin fraset p-piller. Hun har dog for ca. 2 år siden været set af en øjenlæge grundet sløret syn på højre øje. Disse symptomer kom også gradvist men fortog sig indenfor 2 uger.

Du vil gerne udrede patienten på mistanke om Multipel Sklerose. Hvilket af nedenstående udredningsprogrammer er relevant.

a. CT skanning af cerebrum
b. MR skanning af cerebrum og medulla totalis samt lumbalpunktur
c. Elektroneuronografi (ENG) af arme og ben
d. MR af cerebrum
e. Somatosensorisk evokerede potentialer (SSEP)

A

b. MR skanning af cerebrum og medulla totalis samt lumbalpunktur

How well did you know this?
1
Not at all
2
3
4
5
Perfectly
28
Q

Du arbejder i skadestuen og bliver bedt om at tilse en 29 årig kvinde. Hun kommer da hun igennem de seneste dage har fået fornemmelse af tiltagende svigt af venstre ben. Hun er tidligere rask og får ingen medicin fraset p-piller. Hun har dog for ca. 2 år siden været set af en øjenlæge grundet sløret syn på højre øje. Disse symptomer kom også gradvist men fortog sig indenfor 2 uger.
SVAR: Diagnosen Multipel Sklerose baseres på MR skanning og støttes af
lumbalpunkturen.

Lumbalpunkturen viser let pleocytose (8 leukocytter) og forekomst af oligoklonale bånd samt let forhøjet IgG index.
MR skanning af cerebrum (T2 vægtet sekvens) ser således ud:
Er forandringer på ovenstående billede forenelig med diagnosen Multipel Sklerose?
a. Ja
b. Nej

A

a. Ja

How well did you know this?
1
Not at all
2
3
4
5
Perfectly
29
Q

Du arbejder i skadestuen og bliver bedt om at tilse en 29 årig kvinde. Hun kommer da hun igennem de seneste dage har fået fornemmelse af tiltagende svigt af venstre ben. Hun er tidligere rask og får ingen medicin fraset p-piller. Hun har dog for ca. 2 år siden været set af en øjenlæge grundet sløret syn på højre øje. Disse symptomer kom også gradvist men fortog sig indenfor 2 uger.

Svar: Ja, skanningen viser hvid substans læsioner, som radierer ud fra corpus callosum. Disse kaldes for Dawsons finger og er klassiske læsioner ved Multipel Sklerose

Patienten spørger til behandling af hendes aktuelle benparese. Hvilken behandling vil du iværksætte?

a. Højdosis binyrebarkhormon
b. Baklofen
c. Højdosis immunglobulin
d. Plasmaferese
e. Behandling med Tysabri (Natalizumab)

A

a. Højdosis binyrebarkhormon

How well did you know this?
1
Not at all
2
3
4
5
Perfectly
30
Q

Du arbejder i skadestuen og bliver bedt om at tilse en 29 årig kvinde. Hun kommer da hun igennem de seneste dage har fået fornemmelse af tiltagende svigt af venstre ben. Hun er tidligere rask og får ingen medicin fraset p-piller. Hun har dog for ca. 2 år siden været set af en øjenlæge grundet sløret syn på højre øje. Disse symptomer kom også gradvist men fortog sig indenfor 2 uger.

Svar: Til behandling af attaker anvendes højdosis binyrebarkhormon.

Patienten viser sig at have en enægget tvillingesøster (monozygot). Hun er
selvfølgelig meget bange for at hun også udvikler Multipel Sklerose.
Hun spørger der til hvor stor hendes risiko er. Hvad vil du fortælle hende?
a. Risikoen er den samme som for normalbefolkningen
b. Risikoen er ca. 5-7%
c. Risikoen er ca. 25-30%
d. Risikoen er 50%

A

c. Risikoen er ca. 25-30%

How well did you know this?
1
Not at all
2
3
4
5
Perfectly
31
Q

E19

En 70-årig kvinde indlægges på neurologisk afdeling pga akut opståede synsproblemer. Ved neurologisk undersøgelse findes som det eneste neurologiske udfald nedenfor viste synsfeltdefekt, der er fundet ved almindelig klinisk synsfeltundersøgelse. Defekten er angivet som det sorte område.

Hvilken struktur i nervesystemet er læderet?

a. Chiasma opticum
b. Højre occipitallap
c. Højre temporallap
d. Venstre occipitallap
e. Venstre temporallap

A

d. Venstre occipitallap

How well did you know this?
1
Not at all
2
3
4
5
Perfectly
32
Q

E19

60-årig kvinde, tidligere rask, indbringes i FAM på mistanke om apopleksi grundet højresidig hemiparese og afasi med debut for 9 timer siden. Der foretages CT-scanning uden intravenøs kontrast. Hvilket udsagn er korrekt:

a. Normal CT-scanning, således har patienten ikke apopleksi
b. Normal CT-scanning udelukker ikke apopleksi, der bør gives trombolyse
c. Normal CT-scanning udelukker ikke apopleksi, der kan potentielt foretages trombektomi
d. Normal CT-scanning udelukker ikke apopleksi, der kan potentielt gives både trombolyse og foretages trombektomi
e. Der er synligt venstresidigt media infarkt, således ikke indikation for hverken trombolyse eller trombektomi

A

c. Normal CT-scanning udelukker ikke apopleksi, der kan potentielt foretages trombektomi

How well did you know this?
1
Not at all
2
3
4
5
Perfectly
33
Q

E19

72-årig mand indbringes akut i FAM på mistanke om apopleksi grundet pludselig opstået højresidig hemiparese. Der foretages en akut CT-scanning uden intravenøs kontrast. Hvad er den mest sandsynlige diagnose?

a. Intet abnormt
b. Infarkt centralt på venstre side med hæmorhagisk transformation
c. Parenkymblødning centralt på venstre side, formentlig grundet aneurisme. Bør suppleres med CT-angiografi
d. Parenkymblødning centralt på venstre side, formentlig hypertensionsblødning
e. Hyperdens metastase centralt på venstre side

A

d. Parenkymblødning centralt på venstre side, formentlig hypertensionsblødning

How well did you know this?
1
Not at all
2
3
4
5
Perfectly
34
Q

E19

25-årig kvinde går til sin egen læge pga. påvirkning af synet på højre side gennem nogle dage. Synet er sløret, og hun har problemer med farvesyn.

Hvad overvejer du? Hvilken af nedenstående forslag findes mest relevant ?

a. Problem med højre øje, sandsynligvis retina.
b. Læsion i højre occipital lap.
c. Læsion i højre synsnerve
d. Læsion i chiasma.

A

c. Læsion i højre synsnerve

How well did you know this?
1
Not at all
2
3
4
5
Perfectly
35
Q

E19

Hvilke undersøgelser overvejer du?

a. Oftalmoskopi
b. Undersøgelse af farvesyn
c. Undersøgelse af visus
d. Alt ovenstående

A

d. Alt ovenstående

How well did you know this?
1
Not at all
2
3
4
5
Perfectly
36
Q

E19

Du henviser patienten til en øjenlæge, og du foretager neurologisk undersøgelse. Synet er reduceret på højre side, men fundus og den neurologiske undersøgelse er normale. Hvad fortæller du patienten?

a. Du har ikke noget problem, der er ingen sygdom i øjet eller nervesystemet
b. Du har et problem med dine øjne, og du har muligvis brug for nye briller
c. Du har sygdom af nervesystemet, men du ved ikke, hvad det er
d. Du har brug for yderligere undersøgelser

A

d. Du har brug for yderligere undersøgelser

How well did you know this?
1
Not at all
2
3
4
5
Perfectly
37
Q

E19

Du beslutter at undersøge visual evoked responses (VEP). Dette er VEP-resultater (øverste: venstre øje; bund: højre øje). Hvad overvejer du?

a. Der er et forsinket svar (P100-wave) på højre side, og dette tyder på demyelinisering i højre synsnerve
b. Der er forsinket svar (P100-wave) på højre side, og dette indikerer aksonal skade på højre synsnerve
c. Der er et tidligt svar (P100-wave) på venstre side, og dette tyder på demyelinisering i den venstre synsnerve
d. Resultaterne viser normale variation mellem de to sider

A

a. Der er et forsinket svar (P100-wave) på højre side, og dette tyder på demyelinisering i højre synsnerve

How well did you know this?
1
Not at all
2
3
4
5
Perfectly
38
Q

E19

Du beslutter, at der er akut demyelinisering i højre synsnerve, og du stiller diagnosen opticus neurit. Synet er kraftigt reduceret. Hvad vil du gøre

a. Jeg beder patienten om at vente, da det vil forbedres af sig selv
b. Jeg foreslår højdosis methylprednisolon-pulsbehandling
c. Jeg foreslår behandling med betainterferon
d. Jeg foreslår non-steroid antiinflammatoriske lægemidler (NSAID

A

b. Jeg foreslår højdosis methylprednisolon-pulsbehandling

How well did you know this?
1
Not at all
2
3
4
5
Perfectly
39
Q

E19

Hvordan diskuterer du situationen med patienten?

a. Jeg spørger, om hun tidligere har haft lignende symptomer
b. Jeg spørger, om hun har haft andre symptomer, der kan tyde på CNS-sygdom
c. Jeg spørger om CNS-sygdom i familien
d. Alt ovenstående

A

d. Alt ovenstående

How well did you know this?
1
Not at all
2
3
4
5
Perfectly
40
Q

E19

Hun fortæller, at hun havde problem med at skrive sidste år: højre hånd var klodset i et par uger. Hun fortæller også, at hun har mindre problemer med vandladning, hyppige urge. Hvad vil du gøre ?

a. Intet, disse er mindre problemer
b. Jeg bestiller MR-skanning af hjernen
c. Jeg bestiller EMG/ENG-undersøgelse, fordi jeg har mistanke om karpaltunnelsyndrom
d. Jeg bestiller CT-undersøgelse af hjernen

A

b. Jeg bestiller MR-skanning af hjernen

How well did you know this?
1
Not at all
2
3
4
5
Perfectly
41
Q

E19

Du bestiller MR-scanning af cerebrum, og den viser T2-hyperintensitet i den højre synsnerve, men intet andet.

Hvad vil du gøre ?

a. Jeg bestiller MR-scanning af rygmarven
b. Jeg fortæller patienten, at dette bekræfter betændelse i højre synsnerve, men der er ikke andre problemer
c. Jeg fortæller patienten, at hun har multiple sklerose
d. Jeg fortæller patienten, at MR-scanning af hjernen skal gentages om 6 måneder

A

a. Jeg bestiller MR-scanning af rygmarven

How well did you know this?
1
Not at all
2
3
4
5
Perfectly
42
Q

E19

Du bestiller MR-scanning af rygmarven, og den viser dette.

Hvad vil du gøre ?

a. Jeg overvejer en tumor i rygmarven og konsulterer med en neurokirurg
b. Jeg beslutter, at kriterierne for dissemination in time (DIT) og space (DIS) er opfyldte, og at patienten har multiple sklerose (MS)
c. Jeg beslutter, at patienten har brug for en undersøgelse af CSV for at etablere diagnosen af MS
d. Jeg bestiller sensorisk evoked potential (SEP) undersøgelse.

A

b. Jeg beslutter, at kriterierne for dissemination in time (DIT) og space (DIS) er opfyldte, og at patienten har multiple sklerose (MS)

How well did you know this?
1
Not at all
2
3
4
5
Perfectly
43
Q

E19

Hvorfor beslutter du, at kriterierne for DIT og DIS er opfyldt?

a. Patienten havde to læsioner i CNS på to typiske steder bekræftet af MRI
b. Patienten havde to attakker bekræftet af VEP og MRI
c. Patienten havde opticus neurit bekræftet af VEP og også en læsion i den cervikale medulla
d. Alle er korrekte

A

d. Alle er korrekte

How well did you know this?
1
Not at all
2
3
4
5
Perfectly
44
Q

E19

Selvom MS er bekræftet, beslutter du, at du gerne vil undersøge CSV. Hvad forventer du?

a. Pleocytose
b. Forøget protein
c. Nedsat sukker
d. Forøget IgG-indeks og forekomst af oligoklonale bånd ved elektroferese

A

d. Forøget IgG-indeks og forekomst af oligoklonale bånd ved elektroferese

How well did you know this?
1
Not at all
2
3
4
5
Perfectly
45
Q

E19

Hvad er dit næste trin?

a. Jeg foreslår at vente på endnu et attak
b. Jeg foreslår, at patienten tager en anden MR-scanning om 6 måneder for at se, om der er sygdomsaktivitet
c. Jeg begynder at diskutere sygdomsmodificerende behandling (DMT) med patienten
d. Jeg gentager methylprednisolon behandlingen om en måned

A

c. Jeg begynder at diskutere sygdomsmodificerende behandling (DMT) med patienten

How well did you know this?
1
Not at all
2
3
4
5
Perfectly
46
Q

E19

Du er medicinstuderende. I skal snart til eksamen i anatomi. Du er til møde med din læsegruppe, 1 kvinde og 1 mand, begge 23 år gamle. I er bagud med pensum og har derfor arbejdet det meste af natten.

Pludselig siger kvinden mærkelige lyde, stirrer frem for sig for derefter at falde ned fra stolen. Hun har nu fråde om munden og rykvise trækninger i arme og ben. I forsøger at komme i kontakt med hende, men hun svarer ikke. I smertestimulerer hende, men hun reagerer slet ikke, hverken verbalt eller bevægemæssigt. Øjnene er vedvarende lukkede.

Du ringer 112 og siger:

a. Pt tidligere rask. Bevidstløs m GCS 6
b. Pt tidligere rask. Bevidstløs m GCS 3
c. Pt tidligere rask. Bevidstløs m GCS 4

A

b. Pt tidligere rask. Bevidstløs m GCS 3

How well did you know this?
1
Not at all
2
3
4
5
Perfectly
47
Q

E19

Du er medicinstuderende. I skal snart til eksamen i anatomi. Du er til møde med din læsegruppe, 1 kvinde og 1 mand, begge 23 år gamle. I er bagud med pensum og har derfor arbejdet det meste af natten.

Pludselig siger kvinden mærkelige lyde, stirrer frem for sig for derefter at falde ned fra stolen. Hun har nu fråde om munden og rykvise trækninger i arme og ben. I forsøger at komme i kontakt med hende, men hun svarer ikke. I smertestimulerer hende, men hun reagerer slet ikke, hverken verbalt eller bevægemæssigt. Øjnene er vedvarende lukkede.

Ambulance og lægebil kommer hurtigt. Nu er din studiekammerat vågnet noget op. Hun er forvirret og kan ikke bevæge venstre arm. Hvad er den mest sandsynlige diagnose?

a. Intracerebral blødning i venstre hjernehalvdel
b. Subarachnoidalblødning
c. Patologisk forandring i højre hjernehalvdel
d. Hydrocephalus

A

c. Patologisk forandring i højre hjernehalvdel

How well did you know this?
1
Not at all
2
3
4
5
Perfectly
48
Q

E19

I den fælles akut modtagelse tilses din studiekammerat. Hun er nu vågnet helt op. Hun har let hovedpine og fortsat lammelse af venstre arm. Lægen har meget travlt og vil sende hende hjem mhp opfølgning hos egen læge. Du usikker på om det er en god ide og foreslår følgende akutte undersøgelse:

a. EEG på mistanke om epilepsi
b. Doppler undersøgelse af halskar
c. CT skanning af cerebrum

A

c. CT skanning af cerebrum

How well did you know this?
1
Not at all
2
3
4
5
Perfectly
49
Q

E19

Skanningen viser en forandring i relation til højre hjernehalvdel. Pt indlægges i neurologisk regi og får foretaget MR skanning af cerebrum (med kontrast) dagen efter. Skanningen (T2 sekvens og T1 sekvens med kontrast) viser følgende:

Hvad mistænker du?

a. Kronisk subduralt hæmatom på højre side
b. Aneurysme på højre arteria cerebri media
c. Tumor proces i højre frontallap

A

c. Tumor proces i højre frontallap

How well did you know this?
1
Not at all
2
3
4
5
Perfectly
50
Q

E19

Det viser sig at din studiekammerat har en gliøs tumor i højre frontallap. Efter operation får hun diagnosen lavgrads gliom – astrocytom – WHO grad II.

Hvad skønner du hendes gennemsnitlige overlevelse er?

a. 15.6 måneder
b. Mere end 2 år, mindre end 5 år
c. Mere end 5 år

A

c. Mere end 5 år

How well did you know this?
1
Not at all
2
3
4
5
Perfectly
51
Q

F18

En 73-årig mand er kendt med atrieflimren. Han er i digoxinbehandling og i antikoagulationsbehandling med Marevan (Vitamin- K-antagonist). Han indlægges nu akut i neurologisk afdeling på grund af pludselig opstået afasi og højresidig hemiparese. CT-skanning af cerebrum ses ovenfor og viser:

a. Kronisk subduralt hæmatom
b. Tumor cerebri
c. Cerebralt infarkt
d. Haemorragia cerebri
e. Epiduralt hæmatom

A

c. Cerebralt infarkt

How well did you know this?
1
Not at all
2
3
4
5
Perfectly
52
Q

V18

En 29-årig kvinde kommer til neurologisk ambulatorium på grund af neurologiske symptomer, der er opstået for cirka en måned siden og som er blevet langsomt forværrede. Kvinden kan fortælle, at hun tidligere har været fuldstændig rask og der er ikke arvelige sygdomme i familien. De aktuelle problemer er udvikling af dobbeltsyn og tendens til hængende øjenlåg i løbet af dagen. Det er sjældent til stede når hun vågner om morgenen, men kommer i løbet af dagen og er således værst
om aftenen. Ud over disse symptomer føler hun sig også lidt mat og overkommer ikke så meget fysisk som tidligere. Symptomerne aftager ofte hvis hun hviler sig lidt.
Objektiv undersøgelse Alment upåvirket. Sufficient og ubesværet respiration. Kranienerver: Ved
blik fremad er der parallelle øjenakser og let hængende højre øjenlåg. Øjenbevægelser er normale.
Ved blik opad opstår der efter 15 sekunder let adduktion af det højre øje og hængende øjenlåg på begge sider. Pupillerne er egale og lysreagerende. Øvrige kranienervefunktioner er normale.
Motorisk: Der er ingen muskelatrofi. Der er normal kraft i alle 4 ekstremiteter. Der er ingen nedsynkning efter 1 minuts strakt arm test. Reflekser: De dybe reflekser er middellivlige og egale både i overekstremiteter og underekstremiteter. Plantarreflekserne er normale. Sensorisk: Der er normal berørings-, smerte-, temperatur-, og vibrationssans distalt i alle 4 ekstremiteter. Gang: Normal.

Ud fra symptomer og fund overvejer du diagnoser og finder det mest sandsynligt, at der er tale om:

a. Følger efter en blodprop i højre side af hjernestammen
b. Myastenia gravis med okulære symptomer
c. Internukleær oftalmoplegi som følge af dissemineret sklerose
d. Horner syndrom på grund af arteria carotis dissektion på højre side

A

b. Myastenia gravis med okulære symptomer

How well did you know this?
1
Not at all
2
3
4
5
Perfectly
53
Q

V18

En 29-årig kvinde kommer til neurologisk ambulatorium på grund af neurologiske symptomer, der er opstået for cirka en måned siden og som er blevet langsomt forværrede. Kvinden kan fortælle, at hun tidligere har været fuldstændig rask og der er ikke arvelige sygdomme i familien. De aktuelle problemer er udvikling af dobbeltsyn og tendens til hængende øjenlåg i løbet af dagen. Det er sjældent til stede når hun vågner om morgenen, men kommer i løbet af dagen og er således værst
om aftenen. Ud over disse symptomer føler hun sig også lidt mat og overkommer ikke så meget fysisk som tidligere. Symptomerne aftager ofte hvis hun hviler sig lidt.

Du finder det mest sandsynligt, at der er tale om myastenia gravis på grund af de vekslende neurologiske symptomer med forværring ved udtrætning, der udelukkende berører motoriske funktioner.
Din første prioritet med hensyn til undersøgelser er nu :
a. MR-scanning af hjerne, hjernestamme og kranienerver
b. Blodprøve til test for muskelenzym (kreatinkinase) og muskelantistoffer
c. Spinalvæskeundersøgelse for celler, protein og immunglobuliner (IgG-index og elektroferese)
d. Blodprøve til test for acetylcholin-receptor-antistof og neurofysiologisk undersøgelse med henblik på repetitiv stimulation

A

d. Blodprøve til test for acetylcholin-receptor-antistof og neurofysiologisk undersøgelse med henblik på repetitiv stimulation

How well did you know this?
1
Not at all
2
3
4
5
Perfectly
54
Q

V18

En 29-årig kvinde kommer til neurologisk ambulatorium på grund af neurologiske symptomer, der er opstået for cirka en måned siden og som er blevet langsomt forværrede. Kvinden kan fortælle, at hun tidligere har været fuldstændig rask og der er ikke arvelige sygdomme i familien. De aktuelle problemer er udvikling af dobbeltsyn og tendens til hængende øjenlåg i løbet af dagen. Det er sjældent til stede når hun vågner om morgenen, men kommer i løbet af dagen og er således værst
om aftenen. Ud over disse symptomer føler hun sig også lidt mat og overkommer ikke så meget fysisk som tidligere. Symptomerne aftager ofte hvis hun hviler sig lidt.

Acetylcholin-receptor-antistof testen var positiv med en titer-værdi på 75 nmol/l. Den neurofysiologiske undersøgelse skal udføres i dag. Du forventer undersøgelsen vil vise følgende:

a. Decrement (faldende amplitude af motorisk aktionspotentiale) ved repetitiv motor nervestimulation
b. Denerveringsaktivitet i den tværstribede muskulatur
c. Øget varighed af motoriske enhedspotentialer
d. Nedsat motorisk ledningshastighed og forlængede distale motoriske latenser

A

a. Decrement (faldende amplitude af motorisk aktionspotentiale) ved repetitiv motor nervestimulation

How well did you know this?
1
Not at all
2
3
4
5
Perfectly
55
Q

V18

En 29-årig kvinde kommer til neurologisk ambulatorium på grund af neurologiske symptomer, der er opstået for cirka en måned siden og som er blevet langsomt forværrede. Kvinden kan fortælle, at hun tidligere har været fuldstændig rask og der er ikke arvelige sygdomme i familien. De aktuelle problemer er udvikling af dobbeltsyn og tendens til hængende øjenlåg i løbet af dagen. Det er sjældent til stede når hun vågner om morgenen, men kommer i løbet af dagen og er således værst om aftenen. Ud over disse symptomer føler hun sig også lidt mat og overkommer ikke så meget fysisk som tidligere. Symptomerne aftager ofte hvis hun hviler sig lidt.

Den neurofysiologiske undersøgelse viste decrement ved repetitiv motor nerve stimulation.
Diagnosen myastenia gravis er nu sikkert fastslået. Inden du starter behandling af tilstanden
overvejer du om der skal gøres yderligere undersøgelser:
a. Der skal ikke foretages flere undersøgelser
b. Der skal tages blodprøve til bestemmelse af VGCC-antistof (Voltage-Gated-CalciumChannels antistof) til undersøgelse for myastent syndrom
c. Der skal gøres helkrops PET-CT-scanning til undersøgelse for paraneoplastisk tilstand
d. Der skal foretages en CT-scanning af thoraks til undersøgelse for thymom

A

d. Der skal foretages en CT-scanning af thoraks til undersøgelse for thymom

56
Q

V18

En 29-årig kvinde kommer til neurologisk ambulatorium på grund af neurologiske symptomer, der er opstået for cirka en måned siden og som er blevet langsomt forværrede. Kvinden kan fortælle, at hun tidligere har været fuldstændig rask og der er ikke arvelige sygdomme i familien. De aktuelle problemer er udvikling af dobbeltsyn og tendens til hængende øjenlåg i løbet af dagen. Det er sjældent til stede når hun vågner om morgenen, men kommer i løbet af dagen og er således værst om aftenen. Ud over disse symptomer føler hun sig også lidt mat og overkommer ikke så meget fysisk som tidligere. Symptomerne aftager ofte hvis hun hviler sig lidt.

CT-scanning af thoraks var normal og viste ikke tegn på thymom. Du starter nu behandling med:

a. Pyridostigmin (acetylcholin-esterase-hæmmer)
b. Plasmaferese givet en gang om måneden
c. Pyridostigmin (acetylcholin-esterase-hæmmer) og immunmodulation med binyrebarkhormon

A

a. Pyridostigmin (acetylcholin-esterase-hæmmer)

57
Q

V18

En 29-årig kvinde kommer til neurologisk ambulatorium på grund af neurologiske symptomer, der er opstået for cirka en måned siden og som er blevet langsomt forværrede. Kvinden kan fortælle, at hun tidligere har været fuldstændig rask og der er ikke arvelige sygdomme i familien. De aktuelle problemer er udvikling af dobbeltsyn og tendens til hængende øjenlåg i løbet af dagen. Det er sjældent til stede når hun vågner om morgenen, men kommer i løbet af dagen og er således værst om aftenen. Ud over disse symptomer føler hun sig også lidt mat og overkommer ikke så meget fysisk som tidligere. Symptomerne aftager ofte hvis hun hviler sig lidt.

Efter du har startet behandling med acetylcholin-esterase-hæmmer får kvinden det meget bedre og
symptomerne forsvinder faktisk helt i løbet af nogle måneder. Desværre er hun meget generet af
rumlen i maven, mavekneb og diarré.
Hvad kan du gøre:
a. Aftrappe acetylcholin-esterase-hæmmer da tilstanden formentlig er kureret
b. Tillægge en lille dosis atropin for at modvirke autonome bivirkninger til acetylcholinesterase-hæmmer
c. Øge dosis af acetylcholin-esterase-hæmmer da mavegenerne nok er relateret til myastenis gravis

A

b. Tillægge en lille dosis atropin for at modvirke autonome bivirkninger til acetylcholinesterase-hæmmer

58
Q

V18

Som vagthavende på neurologisk afdeling ser du i skadestuen en 35-årig mand med følgende
symptomer: I løbet af de sidste 2 dage har han udviklet tiltagende sløret syn på venstre øje. Han
har smerter bag ved øjet, som forværres når han bevæger øjet til siden. Han føler, at farvesansen
er påvirket i de grøn/blå nuancer på dette øje.
Er dette foreneligt med iskæmisk apoplexia cerebri (blodprop i hjernen) ?
a. Ja
b. Nej

A

b. Nej

59
Q

V18

Som vagthavende på neurologisk afdeling ser du i skadestuen en 35-årig mand med følgende
symptomer: I løbet af de sidste 2 dage har han udviklet tiltagende sløret syn på venstre øje. Han
har smerter bag ved øjet, som forværres når han bevæger øjet til siden. Han føler, at farvesansen
er påvirket i de grøn/blå nuancer på dette øje.
Dette er ikke foreneligt med iskæmisk apoplexia cerebri (blodprop i hjernen) Du har foretaget
oftalmoskopi, og finder på øjet med påvirket syn:
Du stiller nu en tentativ diagnose:
a. Horners syndrom
b. Okulær myastenia gravis
c. Opticus neuritis
d. Retinal migræne
e. Retina løsning

A

c. Opticus neuritis

60
Q

V18

Som vagthavende på neurologisk afdeling ser du i skadestuen en 35-årig mand med følgende symptomer:
I løbet af de sidste 2 dage har han udviklet tiltagende sløret syn på venstre øje. Han har smerter
bag ved øjet, som forværres når han bevæger øjet til siden. Han føler, at farvesansen er påvirket i
de grøn/blå nuancer på dette øje.
Patienten har en opticus neuritis.
Da patienten for 1 år siden havde sensoriske udfald i venstre arm og ben, som remitterede efter 3
uger vælger du at udrede patienten nærmere og får foretaget en MR scanning af hjernen (T1-
vægtet med kontrast):
På baggrund af sygehistorien og MR scanningen stiller du nu diagnosen:
a. Multipel sklerose
b. Encephalitis
c. Neuroborreliose
d. Cerebrale metastaser
e. Multiple cerebrale hæmoragier

A

a. Multipel sklerose

61
Q

V18

Som vagthavende på neurologisk afdeling ser du i skadestuen en 35-årig mand med følgende
symptomer:
I løbet af de sidste 2 dage har han udviklet tiltagende sløret syn på venstre øje. Han har smerter
bag ved øjet, som forværres når han bevæger øjet til siden. Han føler, at farvesansen er påvirket i
de grøn/blå nuancer på dette øje. Du stiller diagnosen multipel sklerose
Patienten ses an. 3 måneder senere møder han med motoriske udfald (parese) af højre arm og
ben. Han har ingen infektion eller feber Du tolker det som et attack som led i hans multipel
sklerose:
Hvilken behandling vil du tilbyde ham for det akutte attak:
a. Penicillin 3 mill IE IV i 7 dage
b. Immunoglobulin behandling 0,2 g hver anden dag i alt 5 dage
c. Plasmaferese hver anden dag i 10 dage
d. Højdosis Prednisolon 500 mg dagligt i 4 dage

A

d. Højdosis Prednisolon 500 mg dagligt i 4 dage

62
Q

V18

Som vagthavende på neurologisk afdeling ser du i skadestuen en 35-årig mand med følgende
symptomer: I løbet af de sidste 2 dage har han udviklet tiltagende sløret syn på venstre øje. Han
har smerter bag ved øjet, som forværres når han bevæger øjet til siden. Han føler, at farvesansen
er påvirket i de grøn/blå nuancer på dette øje. Du vælger at give han højdosis prednisolon i 4 dage
Patienten har haft min. 2 attacker indenfor de sidste 2 år og opfylder dermed kriterier for at
modtage forebyggende behandling mod yderligere attakker. Hvad vil du vælge:
a. Plasmaferese
b. Beta interferon
c. Alfa interferon
d. Kemoterapi
e. Magnyl og persantin

A

b. Beta interferon

63
Q

F18

60 årig kvinde kommer til skadestuen i Åbenrå. Hun og ledsager fortæller at hun har været til fest
– har fået et par genstande alkohol, men føler sig ikke beruset. På vej hjem vil hun stige ind i sin
bil. Idet hun bøjer sig ned besvimer hun. Kommer hurtigt til sig selv igen. Har nu hovedpine.
På skadestuen er patienten helt vågen (GCS 15). Klinisk undersøgelse viser normalt blodtryk,
normal puls, normalt blodsukker. Normalt EKG. Anamnestisk fortæller pt at hun som ung ofte
besvimede. Har ellers været sund og rask fraset let hypertension som er velbehandlet på enkelt
præparat. Indlægges til observation.
Nogle timer senere går pt på toilettet. Kommer ikke tilbage og senere finder personalet hende
liggende på gulvet. Hun er nu tydelig bevidsthedspåvirket. Slår ikke øjnene op på tiltale men på
smertestimulation. Bliver bedt om at sige noget og bevæge sine ekstremiteter. Taler ikke og
bevæger sig ikke på opfordring. På smertestimulation grynter hun og afværger med venstre arm.
Hvad er bedste vurdering af pt’s Glasgow coma score
a. 3
b. 8
c. 14

A

b. 8

64
Q

F18

60 årig kvinde kommer til skadestuen i Åbenrå. Hun og ledsager fortæller at hun har været til fest
– har fået et par genstande alkohol, men føler sig ikke beruset. På vej hjem vil hun stige ind i sin
bil. Idet hun bøjer sig ned besvimer hun. Kommer hurtigt til sig selv igen. Har nu hovedpine.
På skadestuen er patienten helt vågen (GCS 15). Klinisk undersøgelse viser normalt blodtryk,
normal puls, normalt blodsukker. Normalt EKG.
Anamnestisk fortæller pt at hun som ung ofte besvimede. Har ellers været sund og rask fraset let
hypertension som er velbehandlet på enkelt præparat.
Indlægges til observation. Nogle timer senere går pt på toilettet. Kommer ikke tilbage og senere
finder personalet hende liggende på gulvet. Hun er nu tydelig bevidsthedspåvirket. Slår ikke øjnene
op på tiltale men på smertestimulation. Bliver bedt om at sige noget og bevæge sine ekstremiteter.
Taler ikke og bevæger sig ikke på opfordring. På smertestimulation grynter hun og afværger med
venstre arm.
Patienten intuberes og køres til akut CT-scanning af cerebrum.
a. Et infarkt i venstre hemisfære
b. En blødning i subarachnoidalrummet og i parenkymet på venstre side
c. En blødning i subarachnoidalrummet og i parenkymet på højre side
d. Et venstresidigt epiduralt hæmatom
e. Et højresidigt epiduralt hæmatom

A

b. En blødning i subarachnoidalrummet og i parenkymet på venstre side

65
Q

F18

60 årig kvinde kommer til skadestuen i Åbenrå. Hun og ledsager fortæller at hun har været til fest
– har fået et par genstande alkohol, men føler sig ikke beruset. På vej hjem vil hun stige ind i sin
bil. Idet hun bøjer sig ned besvimer hun. Kommer hurtigt til sig selv igen. Har nu hovedpine.
På skadestuen er patienten helt vågen (GCS 15). Klinisk undersøgelse viser normalt blodtryk,
normal puls, normalt blodsukker. Normalt EKG.
Anamnestisk fortæller pt at hun som ung ofte besvimede. Har ellers været sund og rask fraset let
hypertension som er velbehandlet på enkelt præparat.
Indlægges til observation.
Nogle timer senere går pt på toilettet. Kommer ikke tilbage og senere finder personalet hende
liggende på gulvet. Hun er nu tydelig bevidsthedspåvirket. Slår ikke øjnene op på tiltale men på
smertestimulation. Bliver bedt om at sige noget og bevæge sine ekstremiteter. Taler ikke og
bevæger sig ikke på opfordring. På smertestimulation grynter hun og afværger med venstre arm.
Pt overflyttes akut til neurokirurgisk afdeling hvor man udfører ny CT scanning af hjernen.
På en ny scanning vil man se om pt har udviklet komplikationer til blødningen.
Man forventer at tidlig scanning (12 timer efter pt besvimede) vil vise følgende komplikation til
sygdommen
a. Vasospasmer
b. Hydrocephalus
c. Elektrolytforstyrrelser
d. Epilepsi
e. Xantochromi

A

b. Hydrocephalus

66
Q

F18

60 årig kvinde kommer til skadestuen i Åbenrå. Hun og ledsager fortæller at hun har været til fest
– har fået et par genstande alkohol, men føler sig ikke beruset. På vej hjem vil hun stige ind i sin
bil. Idet hun bøjer sig ned besvimer hun. Kommer hurtigt til sig selv igen. Har nu hovedpine.
På skadestuen er patienten helt vågen (GCS 15). Klinisk undersøgelse viser normalt blodtryk,
normal puls, normalt blodsukker. Normalt EKG.
Anamnestisk fortæller pt at hun som ung ofte besvimede. Har ellers været sund og rask fraset let
hypertension som er velbehandlet på enkelt præparat.
Indlægges til observation.
Nogle timer senere går pt på toilettet. Kommer ikke tilbage og senere finder personalet hende
liggende på gulvet. Hun er nu tydelig bevidsthedspåvirket. Slår ikke øjnene op på tiltale men på
smertestimulation. Bliver bedt om at sige noget og bevæge sine ekstremiteter. Taler ikke og
bevæger sig ikke på opfordring. På smertestimulation grynter hun og afværger med venstre arm.
Pt har udviklet hydrocephalus. Får anlagt aflastende eksternt dræn og går straks herefter til
undersøgelse af hjernens blodkar. Denne undersøgelse viser at pt. har to aneurysmer på
henholdsvis ve. arteria cerebri media og højre arteria carotis interna.
Hvilket aneurysme er med størst sandsynlighed ansvarlig for blødningen
a. Venstre arteria cerebri media
b. Højre arteria carotis interna

A

a. Venstre arteria cerebri media

67
Q

F18

Pt får behandlet først det venstresidige og efterfølgende det højresidige aneurysme. Behandlingen
er endovaskulær og der opnås komplet lukning af begge aneurysmer. Efter et par dage er pt
vågen, kan tale og har kun en diskret højresidig hemiparese. Har ikke længere behov for det
eksterne dræn og dette fjernes.
10 dage efter blødningen planlægges pt overflyttet til neurologisk afdeling på Sønderborg Sygehus
men umiddelbart inden ambulancen kommer bliver pt tiltagende sløv og kan ikke bevæge hø. arm.
Der ses let parese af hø. ben. Hun benægter hovedpine og kvalme.
Pt har med størst sandsynlighed udviklet hvilken af nedenstående komplikationer?
a. Hydrocephalus
b. Elektrolytforstyrrelser
c. Vasospasmer
d. Epilepsi

A

c. Vasospasmer

68
Q

E17

En 27-årig venstrehåndet studerende får under løbetur pludselig snurrende
fornemmelser i venstre arm. De snurrende fornemmelser ledsages af små ufrivillige ryk i armen. Han stopper op, ryster armen og generne forsvinder. Han løber videre.
Han har tidligere oplevet noget lignende under eksamen, hvor han tillagde det skrivekrampe. Han slår det derfor hen.
Tre dage senere får han under en forelæsning på seminariet pludselig hovedpine og talebesvær. De snurrende fornemmelser i venstre arm kommer igen. Han har ikke ondt, hverken i armen eller i brystet. Medstuderende kører ham på skadestuen.
Du er, som KBU læge i FAM, den første der ser pt. På det tidspunkt er alt
normaliseret fraset fortsat let hovedpine.
Du bestiller blodprøver inklusiv blodsukker, myocardieprøver og drug screen, samt et EKG. Alle undersøgelser er normale. Du går hen for at tilse anden dårlig patient.
Du bliver akut kaldt tilbage, da patienten nu er sløv og svarer med uforståelig
grynten, når du spørger ham om, hvad der er sket. Han åbner øjnene ved tiltale og
bevæger højresidige ekstremiteter normalt på opfordring.
Du vurderer at pateintens Glasgow Coma Score er:
a. 3
b. 7
c. 11
d. 15

A

*c. 11

69
Q

E17

Du mistænker nu at det drejer sig om en neurologisk problemstilling og tilkalder din
kollega fra neurologisk afdeling. Denne spørger ud til anamnesen. På baggrund af
anamnesen med tidligere tilfælde med snurrende paræstesier i venstre arm tænker
I, at den mest sandsynlige diagnose er
a) cervikal discusprolaps med påvirkning af venstresidig nerverod
b) hjertetilfælde
c) meningitis
a. a
b. b
c. c
d. Ingen af ovenstående

A

*d. Ingen af ovenstående

70
Q

E17

I mistænker cerebral katastrofe og bestiller akut CT-skanning af cerebrum:
Skanningen viser:
a. intracerebral blødning i højre side af storhjernen
b. subarachnoidalblødning i venstre side af storhjernen
c. subarachnoidalblødning i højre side af storhjernen
d. meningeom i venstre side af storhjernen

A

*a. intracerebral blødning i højre side af storhjernen

71
Q

E17

I tilkalder nu neurokirurgisk bagvagt og informerer om, at I har en ung mand med blødning intracerebralt.
I diskuterer mulige årsager til blødningen
a) hjernetumor
b) amfetamin misbrug
c) arteriovenøs malformation
Hvad er mest korrekt ?
a. Ingen af ovenstående årsager

b. Alle ovenstående årsager er mulige
c. Hjernetumor er en meget usandsynlig år bl.a. pga. alderen

A

*b. Alle ovenstående årsager er mulige

72
Q

E17

Pt kommer på neurokirurgisk afdeling. Bedres hurtigt så der bliver ikke behov for intubation eller neurokirurgisk indgreb akut. Han får næste dag foretaget arteriografi, der viser at han har en dybt beliggende arteriovenøs malformaiton= AVM.
Behandling af AVM er:
a. Altid kirurgisk fjernelse
b. Altid endovaskulær lukning
c. Altid strålebehandling
d. Enten kirurgi, endovaskulær lukning eller strålebehandling

A

d. Enten kirurgi, endovaskulær lukning eller strålebehandling

73
Q

F19

100-årig kvinde findes liggende på gulvet ved siden af sin seng af hjemmepleje.
Har bløddelshævelse over kraniet. Der er ingen lateraliserede udfald. Da patienten
er i blodfortyndende behandling foretages CT-scanning som viser:
a. Intet abnormt
b. Højresidigt epiduralt hæmatom med masseeffekt
c. Højresidigt subduralt hæmatom med masseeffekt
d. Højresidig tumor med blødning
e. Højresidigt infarkt med hæmorrhagisk transformation

A

c. Højresidigt subduralt hæmatom med masseeffekt

74
Q

F19

55-årig mand kendt med diabetes og KOL. Vågner om morgenen klokken 8.15
med højresidig hemiparese og tale besvær. Indbringes til OUH, hvor der primært
foretages CT-scanning uden IV-kontrast (se billeder). Hvilket udsagn er mest
sandsynligt ud fra anamnese og fund på CT-scanning?
a. Intet abnormt
b. Subakut venstresidig media infarkt
c. Venstresidig tumor parietalt med masseeffekt
d. Diffust venstresidigt ødem foreneligt med encephalitforandringer
e. Følger af venstresidigt infarkt, men ingen akutte forandringer

A

b. Subakut venstresidig media infarkt

75
Q

F19

Nedenfor er vist en synsfeltdefekt fundet ved almindelig klinisk
synsfeltundersøgelse. Defekten er angivet som det sorte område.
Hvilken struktur i nervesystemet er læderet
a. Chiasma opticum
b. Højre occipitallap
c. Højre parietallap
d. Venstre occiptallap
e. Venstre parietallap

A

a. Chiasma opticum

76
Q

F19

En 21-årig kvinde har månedlig anfald af hovedpine. Hun fortæller at inden
hovedpinen kommer, kan hun ikke se viserne på uret i ca. 20 minutter. Så kommer
hovedpinen med samtidig kvalme og opkastning. Hvilken sygdom overvejer du?
a. Hjernetumor
b. Mutipel sklerose
c. Opticus neurit
d. Migræne med aura

A

d. Migræne med aura

77
Q

F19

En 21-årig kvinde har månedlig anfald af hovedpine. Hun fortæller at inden
hovedpinen kommer, kan hun ikke se viserne på uret i ca. 20 minutter. Så kommer
hovedpinen med samtidig kvalme og opkastning. Hvilken sygdom overvejer du?
Du mistænker migræne med aura. Hun har inden konsultationen fået foretaget MRscanning af hjernen, som viste nogle abnormiteter.
Du får MR billeder:
a. Nej, der er intet symptom på MS.
b. Nej, disse forændringer er ikke typiske for MS, de er typiske for
c. Ja, denne MRC er typisk for MS: periventrikulære læsioner, cerebellare
læsioner og læsioner i corpus callosum
d. Ja, symptomerne kan være MS attak og MR er typisk for MS

A

c. Ja, denne MRC er typisk for MS: periventrikulære læsioner, cerebellare
læsioner og læsioner i corpus callosum

78
Q

F19

En 21-årig kvinde har månedlig anfald af hovedpine. Hun fortæller at inden
hovedpinen kommer, kan hun ikke se viserne på uret i ca. 20 minutter. Så kommer
hovedpinen med samtidig kvalme og opkastning. Hvilken sygdom overvejer du?
MR scanning af cerebrum er typisk for MS og du vil overveje om hun kan have
denne sygdom.
Hvad vil du nu herefter gøre?
a. Jeg fortæller hende at komme tilbage, hvis hun får symptomer igen.
b. Jeg spørger om tidligere symptomer.
c. Jeg fortæller hende, at vi vil gentage MR om 6 måneder.
d. Jeg fortæller hende, at hun også skal have en CT-undersøgelse.

A

b. Jeg spørger om tidligere symptomer.

79
Q

F19

Du spørger om tidligere symptomer, og hun siger, at højre hånd var klodset for
et år siden. Hun havde svært ved at skrive, og havde også en mærkelig følelse og
følelsesløshed i armene i ca. 2 uger. Hun gik ikke til sin læge. Hun havde en
elektrisk følelse i armene, da hun bøjede hovedet. Støtter disse symptomer MS?
a. Ja, det kunne være et attak forårsaget af en læsion i den cervicale
medulla
b. Ja, det kunne være et attak forårsaget af en læsion i den thoracale
medulla
c. Nej, disse symptomer tyder på polyneuropati
d. Nej, disse symptomer tyder på radiculit

A

a. Ja, det kunne være et attak forårsaget af en læsion i den cervicale
medulla

80
Q

F19

Du vurderer, at symptomerne er forenelige med en læsion i den cervikale

medulla. Du undersøger patienten. Hvad forventer du at finde?
a. Øgede abdominale reflekser
b. Reducerede patella og achillesreflekser
c. Atrofi af fingre extensor af begge hænder
d. Ingen af disse
e. Alle disse

A

d. Ingen af disse

81
Q

F19

Du undersøger patienten, og der er ikke noget unormalt ved neurologisk
undersøgelse.
Hvad vil du nu foretage dig?
a. Intet, fordi det udelukker muligheden for MS
b. Jeg fortæller patienten, at hun kunne have MS, men vi behøver ikke at
gøre noget lige nu
c. Jeg fortæller patienten, at hun har MS, og vi skal starte behandling med
det samme
d. Jeg fortæller patienten, at jeg skal bestille yderligere tests

A

d. Jeg fortæller patienten, at jeg skal bestille yderligere tests

82
Q

F19

Du har besluttet dig for at bestille yderligere tests. Hvad bestiller du?

a. MR scanning af medulla totalis
b. CT af thorax
c. CT af cervikal medulla
d. EEG
e. MR scanning af cerebrum

A

a. MR scanning af medulla totalis

83
Q

F19

Du bestiller MR af rygmarven, og det viser en læsion uden kontrastopladning i
cervical medulla.
Hvad er din mening?
a. Det er MS, fordi der er disseminering i sted: Typiske læsioner i to typiske
steder
b. Det er MS, fordi der er disseminering i tid
c. Det er ikke MS, fordi der ikke er disseminering i sted.
d. Det er ikke MS, fordi der ikke er påvist disseminering i tid, selvom der er disseminering i sted

A

d. Det er ikke MS, fordi der ikke er påvist disseminering i tid, selvom der er
disseminering i sted

84
Q

F19

Du bestiller undersøgelse af cerebrospinalvæsken, fordi du skal vise
disseminering i tid.
Hvad forventer du at finde i CSV, hvis du overvejer MS?
a. Forhøjet protein
b. Reduceret glukose
c. Forøget IgG indeks og oligoklonale bånd
d. Reduceret IgG indeks

A

c. Forøget IgG indeks og oligoklonale bånd

85
Q

F19

De oligoklonale bånd og et forhøjet IgG-indeks antyder inflammation i CNS og
disseminering i tid.
Hvad fortæller du patienten?
a. Du har MS, og du skal have en sygdomsmodificerende behandling
b. Du har MS, men du behøver ikke sygdomsmodificerende behandling lige nu
c. Du har MS, og du har brug for methylprednisolonpulsbehandling (500 mg methylprednisolon i 4 dage
d. Du har ikke MS, fordi du kun har haft et klinisk attak

A

a. Du har MS, og du skal have en sygdomsmodificerende behandling

86
Q

E20

En 33-årig kvinde henvises til dig: hun har diffus, pressende hovedpine med kvalme en
gang ugentligt i et år. MR-scanning af cerebrum blev udført og viste følgende:
Hvad viser MR?
a. Det venstre panel viser hypointense-signaler omkring ventriklerne.
b. Det højre panel er normalt, det venstre panel viser signalændringer.
c. Begge er normale.
d. Det venstre panel viser hyperintense signaler omkring ventriklerne, det højre
panel viser sorte huller omkring ventriklerne (black holes).

A

d. Det venstre panel viser hyperintense signaler omkring ventriklerne, det højre

87
Q

E20

En 33-årig kvinde henvises til dig: hun har diffus, pressende hovedpine med kvalme en
gang ugentligt i et år.
Er disse MR-forandringer relateret til hovedpine?
a. Ja, bestemt
b. Ja, nogle af dem
c. Mest sandsynligt ikke
d. Det er ikke muligt at afgøre

A

c. Mest sandsynligt ikke

88
Q

E20

En 33-årig kvinde henvises til dig: hun har diffus, pressende hovedpine med kvalme en
gang ugentligt i et år.
MR forandringerne er formentligt ikke relateret til pt.s hovedpine.
Hvad vil du foretage dig nu?
a. Jeg bestiller straks undersøgelse af CSF.
b. Jeg bestiller en kontrol MR-scanning om 3 måneder.
c. Jeg konsulterer neurokirurger.
d. Jeg spørger om hendes sygehistorie og tidligere symptomer.

A

d. Jeg spørger om hendes sygehistorie og tidligere symptomer.

89
Q

E20

En 33-årig kvinde henvises til dig: hun har diffus, pressende hovedpine med kvalme en
gang ugentligt i et år.
Hun fortæller dig, at hun for to år siden havde sløret syn på højre side. Hvad overvejer du?
a. Intet, det er ikke vigtigt
b. Det kan være opticus neurit
c. Det passer godt til hovedpine, og hun har migræne med visuel aura
d. Hun kan have problemer med retina

A

b. Det kan være opticus neurit

90
Q

E20

En 33-årig kvinde henvises til dig: hun har diffus, pressende hovedpine med kvalme en
gang ugentligt i et år.
Du overvejer opticus neurit på højre side. Hvad vil du spørge om?
a. Havde du smerter under øjenbevægelser?
b. Hvor længe havde du symptomer?
c. Havde du også dobbeltsyn?
d. Jeg spørger både om A og B.

A

d. Jeg spørger både om A og B.

91
Q

E20

En 33-årig kvinde henvises til dig: hun har diffus, pressende hovedpine med kvalme en
gang ugentligt i et år.
Du beslutter, at det understøtter opticus neurit for 2 år siden. Du undersøger patienten, og
pupilreflekserne og visus er normal. Udelukker det opticus neurit?
a. Nej
b. Ja
c. Ja, fordi i de mindste skal visus være reduceret
d. Nej, fordi lysreaktionerne altid er normale, selv når visus er reduceret

A

a. Nej

92
Q

E20

En 33-årig kvinde henvises til dig: hun har diffus, pressende hovedpine med kvalme en
gang ugentligt i et år.
Under den neurologiske undersøgelse finder du let øgede patella-reflekser og nedsat
vibrationssans på store tæer. Er det overraskende?
a. Ja, fordi den ene antyder perifer og den anden CNS-påvirkning.
b. Nej, fordi optikus neurit kan være en del af betændelse af andre perifere nerver.
c. Nej, det kan antyde læsion i medulla spinalis.
d. Ja, fordi patella-reflekser skal være svækkede.

A

c. Nej, det kan antyde læsion i medulla spinalis

93
Q

E20

En 33-årig kvinde henvises til dig: hun har diffus, pressende hovedpine med kvalme en
gang ugentligt i et år
Du har mistanke om multipel sklerose. Du spørger til symptomer relateret til rygmarvslæsion,
men der var ingen i hendes historie. Opfylder hun de diagnostiske kriterier?
a. Ja, fordi der er disseminering i tid og disseminering i sted.
b. Nej, fordi der kun er disseminering i tid.
c. Nej, fordi der kun er disseminering i sted pga. MR-scanning.
d. Nej, fordi der ikke er nogen disseminering i tid eller disseminering i sted

A

*c. Nej, fordi der kun er disseminering i sted pga. MR-scanning.

94
Q

E20

En 33-årig kvinde henvises til dig: hun har diffus, pressende hovedpine med kvalme en
gang ugentligt i et år.
Du har brug for bevis for dissemination in time og optisk neurit. Hvad vil du foretage dig?
a. Jeg bestiller undersøgelse af CSF.
b. Jeg bestiller MR-skanning af rygmarven.
c. Jeg bestiller MR-skanning af rygmarven, undersøgelse af CSF og VEP.
d. Jeg bestiller MEP og SEP

A

c. Jeg bestiller MR-skanning af rygmarven, undersøgelse af CSF og VEP.

95
Q

E20

En 33-årig kvinde henvises til dig: hun har diffus, pressende hovedpine med kvalme en
gang ugentligt i et år.
VEP er normal. MR-scanning af rygmarven viser en enkelt læsion med T2-hyperintenst
signal. CSF viser oligoklonale bånd og forhøjet IgG-indeks. Opfyldes kriterierne for MS?
a. Ja
b. Nej, fordi VEP er normalt.
c. Nej, fordi der burde være flere læsioner i medulla spinalis
d. Ja, fordi VEP altid bliver normal efter heling af optisk neurit i MS

A

a. Ja

96
Q

E20

En 33-årig kvinde henvises til dig: hun har diffus, pressende hovedpine med kvalme en
gang ugentligt i et år.
MS-kriterier er opfyldt. Hvad vil du foretage dig?
a. Jeg giver behandling med højdosis methylprednisolon
b. Jeg bestiller kontrol MR-scanning om 3 måneder.
c. Jeg bestiller kontrol MR-scanning om 6 måneder.
d. Jeg foreslår sygdomsmodificerende behandling

A

*d. Jeg foreslår sygdomsmodificerende behandling

97
Q

F20

72-årig mand med 2 tilfælde af 15 minutters varende let afasi og højresidig armparese. Han tager ikke noget medicin og har ikke noget alkoholoverforbrug.

CT-scanning af hjernen er normal. Udsnit af hans EKG ser således ud:

Hvilken sekundær profylaktisk behandling bør vælges til denne patient?

a. Clopidogrel
b. Vitamin K-antagonist (Marevan) eller NOAK
c. Magnyl
d. Digoxin og magnyl
e. Magnyl, Clopidogrel og vitamin K-antagonist (Marevan)

A

b. Vitamin K-antagonist (Marevan) eller NOAK

98
Q

F20

Du er KBU-læge i modtagelsen og modtager tidligt om morgenen en 73-årig herre, der sidst er set rask aftenen før klokken 22.30. Han er blevet fundet i sengen af hustruen med svær venstresidig hemiparese, da hun har vækket ham med morgenmad på sengen. Efter aftale med radiologen bliver der foretaget en CT-scanning af cerebrum uden IV-kontrast.

Hvad er den mest sandsynlige diagnose ud fra anamnese, klinik og fund på CT-scanning?

a. Intet abnormt, normalvariant med forkalkning i basalganglierne på højre side
b. Parenkymblødning på baggrund af hypertension
c. Iskæmisk infarkt
d. Højresidig tumor
e. Venstresidig tumor

A

b. Parenkymblødning på baggrund af hypertension

99
Q

F20

En 25-årig kvinde klager over snurrende fornemmelser i benene og på abdomen op til ribbenskurvaturen. Det har stået på 1 uge og der er ledsagende urge inkontinens og tunghed i benene.

Hvad viser MR-scanningen ?

a. MR scanningen er normal.
b. Der er en vaskulær læsion i medulla (T2 vægtet)
c. Der er forandring tydende på myelitis
d. Der ses artefakt i medulla

A

c. Der er forandring tydende på myelitis

100
Q

F20

Du modtager en 63-årig herre, der er fundet i hjemmet af sin søn, efter man ikke har haft kontakt til ham i en dags tid. Han er kendt med hypertension men i øvrigt fuldstændig rask. Ved neurologisk undersøgelse finder du ekspressiv og impressiv afasi, apraksi, højresidig central facialis parese samt nedsat kraft i højre overekstremitet. Efter aftale med radiologen foretages CT-scanning uden IV-kontrast.

Hvad er den mest sandsynlige diagnose?

a. Intet abnormt.
b. Følger af venstresidigt infarkt, men intet der kan forklare det aktuelle sygdomsbillede
c. Subakut venstresidigt mediainfarkt
d. Traumatisk betinget ødem
e. Primær venstresidig hjernetumor med omkringliggende ødem

A

c. Subakut venstresidigt mediainfarkt

101
Q

F20

En 78- årig mand er på vandretur på Mallorca. Han får pludselig hovedpine og synsforstyrrelser, der dog hurtigt aftager. Han henvender sig til lokal læge, der finder at han har et for højt blodtryk. Ellers findes intet abnormt. Han anbefales at kontakte egen læge efter hjemkomst til Danmark.

Efter ankomst til Danmark, får han, imens han venter på bagagen i ankomsthallen, pludselig svær hovedpine. Denne hovedpine fortager sig ikke i løbet af dagen, hvorfor hans datter kører ham direkte til skadestuen, hvor du står som ung læge.

Du finder, at han (nu 14 timer efter hovedpinen kom for anden gang) er vågen og har GCS 14, idet han ligger med lukkede øjne. Han er let nakke-rygstiv og ellers neurologisk intakt. Du mistænker subaraknoidalblødning.

Hvilken undersøgelse vil du bestille:

a. MR scanning af cerebrum
b. Lumbalpunktur med undersøgelse arf blodnedbrydningsprodukter
c. Lumbalpunktur med undersøgelse af leukocytter, erytrocytter og protein (rutineundersøgelse)
d. CT scanning af cerebrum

A

d. CT scanning af cerebrum

102
Q

F20

En 78- årig mand er på vandretur på Mallorca. Han får pludselig hovedpine og synsforstyrrelser, der dog hurtigt aftager. Han henvender sig til lokal læge, der finder at han har et for højt blodtryk. Ellers findes intet abnormt. Han anbefales at kontakte egen læge efter hjemkomst til Danmark.

Efter ankomst til Danmark, får han, imens han venter på bagagen i ankomsthallen, pludselig svær hovedpine. Denne hovedpine fortager sig ikke i løbet af dagen, hvorfor hans datter kører ham direkte til skadestuen, hvor du står som ung læge.

Du finder, at han (nu 14 timer efter hovedpinen kom for anden gang) er vågen og har GCS 14, idet han ligger med lukkede øjne. Han er let nakke-rygstiv og ellers neurologisk intakt. Du mistænker subaraknoidalblødning.

CT scanning af cerebrum er normal. Nu vil du anmode om yderligere undersøgelse i form af:

a. MR scanning af cerebrum
b. Lumbalpunktur med undersøgelse af blodnedbrydningsprodukter
c. Lumbalpunktur med virusundersøgelse
d. Ny CT scanning af cerebrum i morgen

A

b. Lumbalpunktur med undersøgelse af blodnedbrydningsprodukter

103
Q

F20

En 78- årig mand er på vandretur på Mallorca. Han får pludselig hovedpine og synsforstyrrelser, der dog hurtigt aftager. Han henvender sig til lokal læge, der finder at han har et for højt blodtryk. Ellers findes intet abnormt. Han anbefales at kontakte egen læge efter hjemkomst til Danmark.

Efter ankomst til Danmark, får han, imens han venter på bagagen i ankomsthallen, pludselig svær hovedpine. Denne hovedpine fortager sig ikke i løbet af dagen, hvorfor hans datter kører ham direkte til skadestuen, hvor du står som ung læge.

Du finder, at han (nu 14 timer efter hovedpinen kom for anden gang) er vågen og har GCS 14, idet han ligger med lukkede øjne. Han er let nakke-rygstiv og ellers neurologisk intakt. Du mistænker subaraknoidalblødning.

Ved lumbalpunktur med undersøgelse for blodnedbrydningsprodukter bekræftes din mistanke om subaraknoidalblødning.

Hvad er den hyppigste årsag til subaraknoidalblødning?

a. Pituitær apopleksi
b. Bristet cerebralt aneurisme
c. Arteriovenøs malformation
d. Blødning i tumor cerebri

A

b. Bristet cerebralt aneurisme

104
Q

F20

En 78- årig mand er på vandretur på Mallorca. Han får pludselig hovedpine og synsforstyrrelser, der dog hurtigt aftager. Han henvender sig til lokal læge, der finder at han har et for højt blodtryk. Ellers findes intet abnormt. Han anbefales at kontakte egen læge efter hjemkomst til Danmark.

Efter ankomst til Danmark, får han, imens han venter på bagagen i ankomsthallen, pludselig svær hovedpine. Denne hovedpine fortager sig ikke i løbet af dagen, hvorfor hans datter kører ham direkte til skadestuen, hvor du står som ung læge.

Du finder, at han (nu 14 timer efter hovedpinen kom for anden gang) er vågen og har GCS 14, idet han ligger med lukkede øjne. Han er let nakke-rygstiv og ellers neurologisk intakt. Du mistænker subaraknoidalblødning.

Den hyppigste årsag til subaraknoidalblødning er bristet cerebralt aneurisme.

Idet du står ved patientens seng og vil fortælle ham om resultatet af undersøgelserne, bliver han fjern. Han svarer ikke længere på tiltale og ved smertestimulation grynter han. Han åbner ikke øjnene, men forsøger at fjerne din hånd fra der hvor du udøver smertestimulation.

Du ringer til neurokirurgisk vagthavende og fortæller om patienten. Du fortæller at han har en Glasgow Coma Scale Score på:

a. 3
b. 6
c. 8
d. 10
e. 12

A

c. 8

105
Q

F20

En 78- årig mand er på vandretur på Mallorca. Han får pludselig hovedpine og synsforstyrrelser, der dog hurtigt aftager. Han henvender sig til lokal læge, der finder at han har et for højt blodtryk. Ellers findes intet abnormt. Han anbefales at kontakte egen læge efter hjemkomst til Danmark.

Efter ankomst til Danmark, får han, imens han venter på bagagen i ankomsthallen, pludselig svær hovedpine. Denne hovedpine fortager sig ikke i løbet af dagen, hvorfor hans datter kører ham direkte til skadestuen, hvor du står som ung læge.

Du finder, at han (nu 14 timer efter hovedpinen kom for anden gang) er vågen og har GCS 14, idet han ligger med lukkede øjne. Han er let nakke-rygstiv og ellers neurologisk intakt. Du mistænker subaraknoidalblødning.

Patienten har en glasgow coma scale score på 8.

Neurokirurgisk vagthavende beder dig om at foretage ny CT scanning.

Billederne - 2 forskellige snit af samme scanning - viser.

Forskellen på den tidligere normale CT scanning og den nye CT scanning er:

a. Tilkomst af SAH
b. Tilkomst af SAH og intracerebralt hæmatom
c. Tilkomst af SAH og epidural blødning
d. Tilkomst af SAH og subduralt hæmatom

A

b. Tilkomst af SAH og intracerebralt hæmatom

106
Q

F17

En 73-årig mand er kendt med atrieflimren. Han er i digoxinbehandling og i antikoagulationsbehandling med Marevan (K-vitaminantagonist). Han indlægges nu akut i neurologisk afdeling på grund af pludselig opstået afasi og højresidig hemiparese. CTskanning af cerebrum ses ovenfor og viser:

  1. Kronisk subduralt hæmatom
  2. Tumor cerebri
  3. Cerebralt infarkt
  4. Haemorragia cerebri
  5. Epiduralt hæmatom
A
  1. Cerebralt infarkt
107
Q

F17

En 35-årig mand henvises med en venstresidig ansigtslammelse. Lammelsen udviklede sig
over et par dage for ca. en uge siden. Ved undersøgelse finder du, at der er påvirkning af
panderynkning, øjenlukning og mundens bevægelser på venstre side. På forespørgsel
oplyser han, at han havde et rødt udslæt på venstre underben for 2 måneder siden. Det så
mærkeligt ud og han tog et billede med sin iPhone.
Udslættet forsvandt af sig selv og han var ikke til lægen i den anledning.
Hvad vil du foretage dig?
1. Bestille en akut CT-scanning af hjernen, da der formentlig er tale om en blodprop
eller en blødning i højre hemisfære
2. Hurtigst muligt få udført en lumbalpunktur med henblik på undersøgelse af spinalvæsken
3. Bestille en MR-scanning af hjernen, da der må være tale om en læsion i
hjernestammen
4. Give kombination af binyrebarkhormon og acyclovir, da det med stor sandsynlighed
er en herpesinfektion

A
  1. Hurtigst muligt få udført en lumbalpunktur med henblik på undersøgelse af spinalvæsken
108
Q

F17

Du er vagthavende læge på neurologisk afdeling og bliver kaldt til neurokirurgisk afdeling for
at vurdere en patient. Det er en 65-årig mand som er kendt med prostata cancer. Han er
blevet indlagt på neurokirurgisk afdeling for 4 timer siden, da han i løbet af dagen udviklede
nedsat kraft i benene og lige før indlæggelsen kunne han ikke længere gå ved egen hjælp.
Han var også generet af summende fornemmelse i benene og han synes også den
underlige fornemmelse strakte sig op på abdomen til omkring navlen.
Neurokirurgerne har fået udført en MR-scanning af rygsøjlen. Billeder ses nedenfor. T1
vægtet til venstre og T2 vægtet til højre.
Neurokirurgerne synes ikke de finder noget der forklarer tilstanden.
Hvad kunne neurokirurgerne diagnostiske mistanke mest sandsynligt have været ?
1. Hæmatom i medulla spinalis niveau TH 10
2. Tumor i knogler niveau TH10 medførende tryk på medulla spinalis
3. Transversel myelitis niveau TH10

A
  1. Tumor i knogler niveau TH10 medførende tryk på medulla spinalis
109
Q

F17

Du er vagthavende læge på neurologisk afdeling og bliver kaldt til neurokirurgisk afdeling for
at vurdere en patient. Det er en 65-årig mand som er kendt med prostata cancer. Han er
blevet indlagt på neurokirurgisk afdeling for 4 timer siden, da han i løbet af dagen udviklede
nedsat kraft i benene og lige før indlæggelsen kunne han ikke længere gå ved egen hjælp.
Han var også generet af summende fornemmelse i benene og han synes også den
underlige fornemmelse strakte sig op på abdomen til omkring navlen.
Neurokirurgerne havde mistænkt knogle metastase fra prostata cancer lokaliseret niveau
TH10.
Du undersøger patienten lidt nøjere og finder, at han også har en let kraftnedsættelse i
armene og de dybe reflekser kan ikke udløses hverken på arme eller ben. Vibrations- og
berøringssansen er nedsat på benene, medens patienten har intakt smerte- og
temperatursans. Du er lidt usikker på om n. facialis funktionen er let påvirket bilateralt.
Hvilken diagnose vil du stille ud fra det kliniske billede ?
1. Spinalis anterior syndrom (blodprop i a. spinalis anterior), i særdeleshed fordi de med
paresen ledsagende sensoriske forstyrrelser passer hermed
2. Infarkt i hjernestammen, fordi der er parese på begge sider og det passer med de
manglende dybe reflekser
3. Guillain Barré syndrom, da tetraparese med ledsagende formodet
kranienervepåvirkning og bortfald af dybe reflekser ikke forklares ved nogen anden
tilstand.
4. Transversel myelitis cervikalt, da MR-scanningen ikke fangede det, da der ikke blev
givet kontrast og cervikal myelitis er i fuld overensstemmelse med bortfald af dybe
reflekser.

A
  1. Guillain Barré syndrom, da tetraparese med ledsagende formodet
    kranienervepåvirkning og bortfald af dybe reflekser ikke forklares ved nogen anden
    tilstand.
110
Q

F17

Du er vagthavende læge på neurologisk afdeling og bliver kaldt til neurokirurgisk afdeling for
at vurdere en patient. Det er en 65-årig mand som er kendt med prostata cancer. Han er
blevet indlagt på neurokirurgisk afdeling for 4 timer siden, da han i løbet af dagen udviklede
nedsat kraft i benene og lige før indlæggelsen kunne han ikke længere gå ved egen hjælp.
Han var også generet af summende fornemmelse i benene og han synes også den
underlige fornemmelse strakte sig op på abdomen til omkring navlen.
Du stiller diagnosen Guillain Barré syndrom.
Du bliver ved supplerende spørgsmål til patienten klar over, at han have en uge varende
diarré for et par uger siden. Det gik over af sig selv og blev ikke nærmere undersøgt. Det
understøtter din diagnostiske antagelse.
Du gør dig lidt overvejelser over hvordan det vil gå patienten på kort og lang sigt.
Hvad er mest korrekt ?
1. Der er stor risiko for, at patienten ender i respirator og kan efterlades med varig
parese
2. Det er usandsynligt, at patienten får behov for respirator og han vil hurtigt komme på
benene igen.
3. Patienten vil måske få behov for respirator, men langtidsprognosen er rigtig god
4. Uden parakliniske undersøgelser kan vi ikke vurdere prognosen

A
  1. Der er stor risiko for, at patienten ender i respirator og kan efterlades med varig
    parese
111
Q

F17

Du er vagthavende læge på neurologisk afdeling og bliver kaldt til neurokirurgisk
afdeling for at vurdere en patient. Det er en 65-årig mand som er kendt med prostata
cancer. Han er blevet indlagt på neurokirurgisk afdeling for 4 timer siden, da han i løbet
af dagen udviklede nedsat kraft i benene og lige før indlæggelsen kunne han ikke
længere gå ved egen hjælp. Han var også generet af summende fornemmelse i benene
og han synes også den underlige fornemmelse strakte sig op på abdomen til omkring
navlen.
På baggrund af alder, forudgående diarré og den hurtige initiale udvikling er der stor
risiko for at patienten får behov for respirator og at han efterlades med en vis grad af
varig parese.
Hvilke yderligere undersøgelser er vigtigst ?
1. MR-scanning af hjernen og nerveledning
2. Blodprøver for antistoffer og nerveledning
3. Muskel- og nervebiopsi for at se efter inflammation
4. Spinalvæske undersøgelse for protein og celler, samt nerveledning

A
  1. Spinalvæske undersøgelse for protein og celler, samt nerveledning
112
Q

F17

Du er vagthavende læge på neurologisk afdeling og bliver kaldt til neurokirurgisk afdeling for
at vurdere en patient. Det er en 65-årig mand som er kendt med prostata cancer. Han er
blevet indlagt på neurokirurgisk afdeling for 4 timer siden, da han i løbet af dagen udviklede
nedsat kraft i benene og lige før indlæggelsen kunne han ikke længere gå ved egen hjælp.
Han var også generet af summende fornemmelse i benene og han synes også den
underlige fornemmelse strakte sig op på abdomen til omkring navlen.
Du finder spinalvæske undersøgelse for protein og celler, samt nerveledning vigtigst.
Efter patienten er flyttet til neurologisk afdeling overvejer du videre tiltag.
Hvad er det mest korrekte at gøre nu ?
1. Der skal indledes trombose profylakse, tæt overvågning af respiration inklusiv måling
af vital kapacitet, og højdosis intravenøs immunglobulin
2. Patienten observeres og behandles kun symptomatisk, herunder respirator ved
behov
3. Hvis der gives højdosis intravenøs immunglobulin, behøver man ikke give trombose
profylakse og når vi kommer i gang så hurtigt, er der ikke risiko for svigt af
respirationen.

A
  1. Der skal indledes trombose profylakse, tæt overvågning af respiration inklusiv måling
    af vital kapacitet, og højdosis intravenøs immunglobulin
113
Q

F17

Du er vagthavende læge på neurologisk afdeling og bliver kaldt til neurokirurgisk afdeling for
at vurdere en patient. Det er en 65-årig mand som er kendt med prostata cancer. Han er
blevet indlagt på neurokirurgisk afdeling for 4 timer siden, da han i løbet af dagen udviklede
nedsat kraft i benene og lige før indlæggelsen kunne han ikke længere gå ved egen hjælp.
Han var også generet af summende fornemmelse i benene og han synes også den
underlige fornemmelse strakte sig op på abdomen til omkring navlen.
Du vælger at observere tæt med måling med bl.a. måling af vitalkapacitet, indleder
trombose profylakse, samt giver højdosis intravenøs immunglobulin.
Dagen efter indlæggelsen får du resultaterne af spinalvæske undersøgelse og
nerveledning.
Spinalvæsken har normalt celletal og protein indhold og nerveledningen viser lidt
uspecifikke forandringer herunder forlængede F-wave latencer.
Hvad er den mest korrekte efter disse svar ?
1. Med normale undersøgelser kan det ikke være Guillain Barré syndrom
2. Guillain Barré syndrom er usandsynligt men ikke udelukket
3. Diagnosen Guillain Barré syndrom fastholdes da undersøgelserne ofte er normale
tidligt i forløbet

A
  1. Diagnosen Guillain Barré syndrom fastholdes da undersøgelserne ofte er normale
    tidligt i forløbet
114
Q

F17

Du er vagthavende læge en aften på en akutmodtagelse. Der ankommer en akut patient,
indlagt via lægevagten. Det er en 82-årig mand, der bor i eget hjem med højt
funktionsniveau. Tidligere har han fået skiftet hø. hofte og er blevet behandlet på
hjerteafdelingen med en stent. Han er derfor i behandling med Clopidogrel 75mg x 1 og
Hjertemagnyl 75mg x 1.
For en måned siden havde han en faldepisode med let hovedtraume og hanblev syet med 4
sting i skadestuen. Kom sig godt oven på det, men gennem den seneste uge har han haft
nedsat kraft i hø. arm og hø. ben med snigende debut. Her til aften er han meget træt, svær
af vække.
Baseret på ovenstående sygehistorie, hvilken sygdom er den mest sandsynlige?
1. Intracerebralt hæmatom
2. Tumor Cerebri
3. Kronisk Subduralt Hæmatom
4. Subarachnoidalblødning

A
  1. Kronisk Subduralt Hæmatom
115
Q

F17

Du er vagthavende læge en aften på en akutmodtagelse. Der ankommer en akut patient,
indlagt via lægevagten. Det er en 82-årig mand, der bor i eget hjem med højt
funktionsniveau. Tidligere har han fået skiftet hø. hofte og er blevet behandlet på
hjerteafdelingen med en stent. Han er derfor i behandling med Clopidogrel 75mg x 1 og
Hjertemagnyl 75mg x 1.
For en måned siden havde han en faldepisode med let hovedtraume og hanblev syet med 4
sting i skadestuen. Kom sig godt oven på det, men gennem den seneste uge har han haft
nedsat kraft i hø. arm og hø. ben med snigende debut. Her til aften er han meget træt, svær
af vække.
Du finder det mest sandsynligt, at patienten har kronisk subduralt hæmatom.
Du undersøger patienten og finder, at han akkurat kan vækkes til at svare enkelte ord, men
falder hurtigt i søvn igen. Han bevæger ikke højre arm eller ben spontant, på opfordring eller
på smertestimulation.
Hvilket scoresystem bruges typisk til at vurdere bevidsthedsniveau hos en patient under de
omstændigheder?
1. Glasgow Coma Scale (GCS)
2. Advanced Traume Life Support (ATLS)
3. Critical Neurology Alertness Scale (CNA)
4. Brain Function Monitor Scale (BFM)

A
  1. Glasgow Coma Scale (GCS)
116
Q

F17

Du er vagthavende læge en aften på en akutmodtagelse. Der ankommer en akut patient,
indlagt via lægevagten. Det er en 82-årig mand, der bor i eget hjem med højt
funktionsniveau. Tidligere har han fået skiftet hø. hofte og er blevet behandlet på
hjerteafdelingen med en stent. Han er derfor i behandling med Clopidogrel 75mg x 1 og
Hjertemagnyl 75mg x 1.
For en måned siden havde han en faldepisode med let hovedtraume og hanblev syet med 4
sting i skadestuen. Kom sig godt oven på det, men gennem den seneste uge har han haft
nedsat kraft i hø. arm og hø. ben med snigende debut. Her til aften er han meget træt, svær
af vække.
Til vurdering af bevidsthedsniveau anvendes Glasgow Coma Scale (GCS)
Der foretages en CT-scanning i akutmodtagelsen.
Forklarer denne scanning patientens symptombillede?
1. Ja
2. Nej

A
  1. Ja
117
Q

F17

Du er vagthavende læge en aften på en akutmodtagelse. Der ankommer en akut patient,
indlagt via lægevagten. Det er en 82-årig mand, der bor i eget hjem med højt
funktionsniveau. Tidligere har han fået skiftet hø. hofte og er blevet behandlet på
hjerteafdelingen med en stent. Han er derfor i behandling med Clopidogrel 75mg x 1 og
Hjertemagnyl 75mg x 1.
For en måned siden havde han en faldepisode med let hovedtraume og hanblev syet med 4
sting i skadestuen. Kom sig godt oven på det, men gennem den seneste uge har han haft
nedsat kraft i hø. arm og hø. ben med snigende debut. Her til aften er han meget træt, svær
af vække.
CT-scanningen forklarer tilstanden og bekræfter diagnosen kronisk subduralt hæmatom.
På grund af patientens dårlige bevidsthedsniveau besluttes at iværksætte akut behandling
med formål at sænke det intrakranielle tryk.
Hvilken behandling vil du iværksætte akut?
1. Sænke Blodtryk til under 110 systolisk
2. Hyperton KCl
3. Administrere betablokker I.V.
4. Administrere morfin I.V.
5. Intubere og sedere patienten

A
  1. Intubere og sedere patienten
118
Q

F17

Afventende en transport til Neurokirurgisk afdeling, overvejer du at behandle med stereoid.
Hvilke af følgende ødemtilstande i hjernen behandles typisk IKKE med stereoid?
1. Tumor-induceret ødem (f.eks. meningeom)
2. Traume-induceret ødem (f.eks. akut subduralt hæmatom)
3. Infektions-induceret ødem (f.eks. meningitis)

A
  1. Traume-induceret ødem (f.eks. akut subduralt hæmatom)
119
Q

F17
Patienten bliver opereret og viser sig at have et vedvarende behov for at lede
cerebrospinalvæsken væk fra hjernen. Derfor anlægges en Shunt.
Hvilken anatomisk lokalisation er normalt førstevalget at lede cerebrospinalvæsken til via en
Shunt?
1. Subarachnoidalrummet
2. Pleura
3. Sinus Transversus
4. Via vene jugularis til hø. atrie
5. Peritoneum

A
  1. Peritoneum
120
Q

E16

En 76-årig tidligere rask kvinde har gennem det seneste ½ år haft lettere intermitterende
hovedpine. Nu indlægges hun på grund af pludselig opstået venstresidig hemiparese og
bevidsthedspåvirkning. Ledsagende er der hovedpine, kvalme og opkastninger. Ved
indlæggelsen er blodtryk 220/130 mmHg. EKG viser sinusrytme og venstresidig hypertrofi.
Akut CT-skanning af cerebrum ses ovenfor og viser:
1. Multipel sklerose
2. Kronisk subduralt hæmatom
3. Epiduralt hæmatom
4. Hæmorhagia cerebri/Intracerebralt hæmatom
5. Infarctus cerebri/Iskæmisk apopleksi

A
  1. Hæmorhagia cerebri/Intracerebralt hæmatom
121
Q

E16

Essay 6 – delopgave 1
58 årig kvinde kommer til FAM pga. pludselig opstået lammelse i højre ansigtshalvdel og arm, samt talebesvær. Ledsagende ægtefælle oplyser at talen er blevet bedre siden symptomerne opstod for 1 time siden. Patienten er vågen og virker orienteret. Patienten har tidligere fået en ny hjerteklap og er i behandling med blodfortyndende medicin = marevan. Dette styres via jævnlige blodprøver hos egen læge. Hun har taget sin medicin som vanligt og har ikke været alment syg i perioden op til aktuelle. Hun blev i
forrige uge opereret med indsættelse af ny hofte. Forløbet var ukompliceret.
Du vælger at kontakte trombolyse vagthavende på afdeling N. Denne er pga akut aktivitet ikke i stand til at give møde og du bliver bedt om at bestille relevant undersøgelsesprogram. Du bestiller udover blodprøver (INR ligger lavt i terapeutisk niveau) og EKG :
1. Lumbal punktur på mistanke om subarachnoidalblødning
2. MR skanning af cerebrum på mistanke om hjernetumor
3. CT skanning af cerebrum på mistanke om blodprop i hjernen og for samtidig at udelukke hjerneblødning
4. Afventer at bestille undersøgelse idet du forventer at det drejer sig om TCI (transitorisk cerebral iskæmi) tilfælde

A
  1. CT skanning af cerebrum på mistanke om blodprop i hjernen og for samtidig at udelukke hjerneblødning
122
Q

E16

Essay 6 – delopgave 2
58 årig kvinde kommer til FAM pga. pludselig opstået lammelse i højre ansigtshalvdel og arm, samt talebesvær. Ledsagende ægtefælle oplyser at talen er blevet bedre siden symptomerne opstod for 1 time siden. Patienten er vågen og virker orienteret.
Patienten har tidligere fået en ny hjerteklap og er i behandling med blodfortyndende medicin = marevan. Dette styres via jævnlige blodprøver hos egen læge. Hun har taget sin medicin som vanligt og har ikke været alment syg i perioden op til aktuelle. Hun blev i forrige uge opereret med indsættelse af ny hofte. Forløbet var ukompliceret. Du har bestilt en CT scanning af cerebrum og den viser (med størst sandsynlighed):
1. en blødning i venstre side af hjernen
2. et infarkt i venstre side af hjernen
3. en tumor i venstre side af hjernen
4. hydrocephalus

A
  1. et infarkt i venstre side af hjernen
123
Q

E16

Essay 6 – delopgave 3
58 årig kvinde kommer til FAM pga. pludselig opstået lammelse i højre ansigtshalvdel og arm, samt talebesvær. Ledsagende ægtefælle oplyser at talen er blevet bedre siden symptomerne opstod for 1 time siden. Patienten er vågen og virker orienteret. Patienten har tidligere fået en ny hjerteklap og er i behandling med blodfortyndende medicin = marevan. Dette styres via jævnlige blodprøver hos egen læge. Hun har taget sin medicin som vanligt og har ikke været alment syg i perioden op til aktuelle. Hun blev i forrige uge opereret med indsættelse af ny hofte. Forløbet var ukompliceret.
Skanningen giver mistanke om en stor blodprop i venstre hjernehalvdel. Der er nu gået et par timer efter patienten kom på sygehuset. Patienten er vurderet ikke at være kandidat til trombolysebehandling og det har ikke været muligt at foretage fjernelse af en stor trombe i venstre arteria cerebri media.
Patienten går på neurologisk afdeling til observation. Nu taler hun næsten normalt men har uændret lammelse i højre ansigtshalvdel og arm. Udover behandling og genoptræning start skal hun observeres på mistanke om udvikling af bevidsthedtab.
Et evt efterfølgende fald i bevidsthedsniveau vil højest sandsynlig skyldes forhøjet tryk intrakranielt på baggrund i :
1. udvikling af hjerneblødning idet patienten er i blodfortyndende behandling
2. udvikling af hydrocephalus
3. udvikling af ødem omkring det store infarkt medførende ’malignt mediainfarkt’.

A
  1. udvikling af ødem omkring det store infarkt medførende ’malignt mediainfarkt’.
124
Q

E16

Essay 6 – delopgave 4
58 årig kvinde kommer til FAM pga. pludselig opstået lammelse i højre ansigtshalvdel og arm, samt talebesvær. Ledsagende ægtefælle oplyser at talen er blevet bedre siden symptomerne opstod for 1 time siden. Patienten er vågen og virker orienteret. Patienten har tidligere fået en ny hjerteklap og er i behandling med blodfortyndende medicin = marevan. Dette styres via jævnlige blodprøver hos egen læge. Hun har taget sin medicin som vanligt og har ikke været alment syg i perioden op til aktuelle. Hun blev i
forrige uge opereret med indsættelse af ny hofte. Forløbet var ukompliceret.
Næste morgen er patienten svær at vække og neurologisk undersøgelse viser uændret højresidig ansigts- og armparalyse. Der er tilkommet lammelse af højre ben. Patienten taler slet ikke og følger ikke opfordringer. Hun åbner øjne på høj tiltale og smertestimuleres hun siger hun av og forsøger med den raske arm at afværge. Du kender ikke den neurologiske vurderingsskala og vurderer derfor patienten efter Glasgow Coma Scala til at have en score på
1. 1
2. 4
3. 10
4. 15

A
  1. 10
125
Q

E16

Essay 6 – delopgave 5
58 årig kvinde kommer til FAM pga. pludselig opstået lammelse i højre ansigtshalvdel og arm, samt talebesvær. Ledsagende ægtefælle oplyser at talen er blevet bedre siden symptomerne opstod for 1 time siden. Patienten er vågen og virker orienteret.
Patienten har tidligere fået en ny hjerteklap og er i behandling med blodfortyndende medicin = marevan. Dette styres via jævnlige blodprøver hos egen læge. Hun har taget sin medicin som vanligt og har ikke været alment syg i perioden op til aktuelle. Hun blev i forrige uge opereret med indsættelse af ny hofte. Forløbet var ukompliceret.
Du bestiller nu akut CT skanning af cerebrum og denne viser at der er tale om malignt mediainfarkt. Du kontakter neurokirurgisk vagthavende der finder indikation for at behandle patienten.
Neurokirurgisk behandling består primært af:
1. operation med fjernelse af en stor del af kranieknoglen
2. anlæggelse af eksternt dræn til aflastning for væske i ventrikelsystemet
3. anlæggelse af spinal kateter til trykmåling
4. neurointensiv behandling m sedation, intubation og hyperton NaCl

A
  1. operation med fjernelse af en stor del af kranieknoglen
126
Q

E20

50-årig tidligere rask kvinde indbringes akut i FAM grundet akut opstået hovedpine og aftagende bevidsthedsniveau. Der foretages en akut CT-scanning uden IV-kontrast. Hvad er det mest korrekte udsagn?

a. Intet abnormt.
b. Subduralt hæmatom
c. Parenkymatøs hypertensionsblødning
d. Spontan SAH, formentlig grundet aneurisme. SAH, formentlig traumatisk betinget

A

d. Spontan SAH, formentlig grundet aneurisme. SAH, formentlig traumatisk betinget

127
Q

E20

44-årig mand, ikke kendt sygdom fra tidligere. Findes mandag formiddag af pårørende med nedsat bevidsthed. Har ikke været i kontakt med pårørende hele weekenden. Intuberes og indbringes til FAM, hvor der foretages CT-scanning uden IV-kontrast. Hvad er den mest sandsynlige diagnose?

a. Intet abnormt
b. Venstresidigt a. cerebri media infarkt med masseeffekt
c. Følger af tidligere venstresidigt infarkt uden relevans for aktuelle sygdomsbillede
d. Venstresidig blødning med ødem og masseeffekt
e. Venstresidig tumor med udtalt ødem

A

b. Venstresidigt a. cerebri media infarkt med masseeffekt

128
Q

V18

6-årig pige indbringes med Falck til skadestuen med pludseligt indsættende kramper. Subfebril. Kramperne ligner ikke typiske feberkramper. Der bestilles CT-skanning af cerebrum uden intravenøst kontrast.
Hvad viser CT-skanningen?

a. Intet abnormt
b. Udtalt forkalkning i hjerneparenkymet i frontale hjerneafsnit
c. Stor frisk intrakraniel blødning i frontale hjerneafsnit
d. Stor malign tumor i frontale hjerneafsnit
e. Friske infaktforandringer svarende til a. cerebri anterior på begge sider

A

c. Stor frisk intrakraniel blødning i frontale hjerneafsnit

129
Q

V18

En 35-årig mand henvises med en venstresidig ansigtslammelse. Lammelsen udviklede sig over et par dage for ca. en uge siden. Ved undersøgelse finder du, at der er påvirkning af panderynkning, øjenlukning og mundens bevægelser på venstre side. På forespørgsel oplyser han, at han havde et rødt udslæt på venstre underben for 2 måneder siden. Det så mærkeligt ud og han tog et billede med sin iPhone.
[BILLEDE]
Udslættet forsvandt af sig selv og han var ikke til lægen i den anledning.
Hvad vil du foretage dig?

a. Bestille en akut CT-scanning af hjernen, da der formentlig er tale om en blodprop eller en blødning i højre hemisfære
b. Hurtigst muligt få udført en lumbalpunktur med henblik på undersøgelse af spinalvæsken
c. Bestille en MR-scanning af hjernen, da der må være tale om en læsion i hjernestammen
d. Give kombination af binyrebarkhormon og acyclovir, da det med stor sandsynlighed er en herpesinfektion

A

b. Hurtigst muligt få udført en lumbalpunktur med henblik på undersøgelse af spinalvæsken

130
Q

V18

39-årig kvinde indbringes med Falck til skadestuen med pludselig indsættende hovedpine. Hovedpinen kom under klipning af hæk, muligvis faldet efterfølgende. Født med læbe-gane- spalte. Lyssky. Subfebril. Nakke-rygstiv. Meget sløv og sovende. Opkastninger og kvalme. Der bestilles CT-skanning af cerebrum uden intravenøst kontrast.
[BILLEDE]
Hvad viser CT scanningen?

a. Intet abnormt
b. Fortykket tentorium
c. Friske bilaterale subdurale blødninger
d. Frisk subarachnoidal blødning
e. Frisk intracerebral blødning

A

d. Frisk subarachnoidal blødning

131
Q

E20

Der mistænkes cerebral apopleksi hvorfor der foretages akut CT-cerebrum.
[SCANNING]
Scanningen viser en mindre blødning i basalganglierne på højre side. Hyppigste årsag til spontan intracerebral blødning er:

a. Bristet aneurysme
b. Forhøjet blodtryk
c. Atrieflimren
d. Koagulopati
e. Stofmisbrug

A

b. Forhøjet blodtryk

132
Q

E17

En 76-årig tidligere rask kvinde har gennem det seneste ½ år haft lettere intermitterende hovedpine. Nu indlægges hun på grund af pludselig opstået venstresidig hemiparese og bevidsthedspåvirkning. Ledsagende er der hovedpine, kvalme og opkastninger. Ved indlæggelsen er blodtryk 220/130 mmHg. EKG viser sinusrytme og venstresidig hypertrofi. Akut CT-skanning af cerebrum ses ovenfor og viser:

a. Multipel sklerose
b. Kronisk subduralt hæmatom
c. Epiduralt hæmatom
d. Hæmorhagia cerebri/Intracerebralt hæmatom
e. Infarctus cerebri/Iskæmisk apopleksi

A

d. Hæmorhagia cerebri/Intracerebralt hæmatom

133
Q

E17

72-årig mand med 2 tilfælde af 15 minutters varende let afasi og højresidig armparese. Han tager ikke noget medicin og har ikke noget alkoholoverforbrug. CT-scanning af hjernen er normal.
Udsnit af hans EKG ser således ud:
[BILLEDE]
Hvilken sekundær profylaktisk behandling bør vælges til denne patient?

a. Magnyl og dipyridamol (persantin)
b. Vitamin K-antagonist (Marevan)
c. Magnyl
d. Digoxin og magnyl
e. Magnyl, dipyridamol (persantin) og vitamin K-antagonist (Marevan)

A

b. Vitamin K-antagonist (Marevan)

134
Q

E17

En 55-årig mand har den seneste måned haft gentagne anfald, hvor han bliver fjern og foretager automathandlinger. Anfaldene er af ca. 5 minutters varighed og efterfølges af desorientering i ca. 15 minutter. Anfaldene er stereotype. Ved den neurologiske undersøgelse findes en synsfeltdefefekt som vist nedenfor. MR-scanning af cerebrum har vist en tumor. Hvad er lokalisationen af tumor?
[BILLEDE]

a. Hypofysen
b. Højre temporallap
c. Venstre occipitallap
d. Højre occipitallap
e. Venstre temporallap

A

b. Højre temporallap

135
Q
A